托业考试阅读模拟题

合集下载

托业(阅读)模拟试卷36(题后含答案及解析)

托业(阅读)模拟试卷36(题后含答案及解析)

托业(阅读)模拟试卷36(题后含答案及解析) 题型有:1. 完成句子 2. 短文填空 3. 阅读理解完成句子1.Anyone who enters the factory floor should wear a helmet and safety vest at ______ times.A.anyB.allC.everyD.each正确答案:B解析:at all times一直,始终2.The speaker decided to ______ talk about the company’s history before explaining his plans for its future.A.briefB.briefsC.brieflyD.briefing正确答案:C解析:副词位置3.The organizers expect between 40,000 ______ 50,000 people to attend the day-long music festival.A.orB.andC.bothD.either正确答案:B解析:between A and B(在A与B之间)4.______ information about the department’s expenditures is available in the monthly report.A.DetailB.DetailsC.DetailingD.Detailed正确答案:D解析:形容词位置,过去分词5.Due to mechanical problems, there will be a two-hour ______ to the departure of Flight 309.A.delayB.pauseC.cancellationD.stopover正确答案:A解析:delay延期,耽搁6.The Daily Star sent a journalist to Iran to ______ on the election in that country.A.accountB.informC.reportD.cover正确答案:C解析:report on就……作报告7.Beginning March 31, Western Service will regularly ______ two new bus routes along Hathfield Avenue.A.operateB.operatingC.operatedD.operative正确答案:A解析:动词位置,<助动词+动词原形>8.______ company executives and union members can reach an agreement, the workers will go on strike.A.IfB.WhenC.UnlessD.After正确答案:C解析:条件连词unless(除……之外)9.Ms. Corbett’s generous ______ gave the organization enough money to builda new community hall.A.attributeB.effortC.assetD.donation正确答案:D解析:donation(主要是金钱上的)捐赠10.A growing ______ of countries have signed the international agreement to reduce emissions of greenhouse gases.A.fileB.numeralC.listD.log正确答案:C解析:list目录,清单11.To become skilled in martial arts like karate requires extensive ______ and constant dedication.A.trainB.trainedC.trainingD.to train正确答案:C解析:名词位置,training(训练,教养)12.Visitors to the Canadian History Museum should use the side entrance while the front entrance ______.A.is renovatingB.is being renovatedC.has renovatedD.has been renovated正确答案:B解析:时态,语态13.Industry analysts were surprised by how ______ the economy recovered from recession.A.quickB.quicklyC.quickerD.quicken正确答案:B解析:副词位置14.Passengers with over 20 kilograms of luggage will have to pay an ______ fee of $10 per kilogram.A.accessibleB.inevitableC.irrationalD.additional正确答案:D解析:additional附加的15.A good manager has the ability to take ______ of any situation and help the employees to focus on their own tasks.A.feeB.betC.dutyD.charge正确答案:D解析:take charge of接管,负责16.The incoming chief executive knows the actions ______ need to be done to turn around the company’s performance.A.whyB.thatC.whoD.what正确答案:B解析:可替代人或事物等先行词的关系代词that17.When the budget is announced, Mr. Watanabe will determine whether to add another employee to ______ team.A.heB.hisC.himD.himself正确答案:B解析:代词的所有格18.No one else ______ the building janitor has access to Mr. McKenzie’s office on the 10th floor.A.withoutB.exceptC.fromD.on正确答案:B解析:介词except(除……之外)19.All work stations should be properly ______ with a desk, computer, monitor, lamp, and telephone.A.equipB.equippedC.equippingD.to equip正确答案:B解析:被动的过去分词20.Bisworth Travel customers can adjust their booking at ______ extra cost provided that they do so at least 7 days prior to departure.A.noneB.notC.noD.never正确答案:C解析:否定形容词no(完全没有)21.The company ______ a new policy on employee Internet usage at the start of the year.A.formattedB.coinedC.implementedD.ascribed正确答案:C解析:implement实施(制度,政策,决定等)22.Ms. Rogers ______ failed to carry out her duties in a professional and acceptable manner.A.believablyB.impossiblyC.repeatedlyD.realistically正确答案:C解析:repeatedly再三地23.The decision ______ the fishing industry more strictly came about because of concerns about the marine environment.A.regulatedB.regulatesC.to regulateD.is regulating正确答案:C解析:形容词位置,不定词to的形容词用法24.To reach N-Tech’s head office, turn left at Godfrey Lake and keep going ______ the botanical gardens.A.withB.forC.pastD.on正确答案:C解析:地点介词past(过某处)25.The Australian company Core Ltd. is the ______ exporter of coal and iron in the southern hemisphere.A.largeB.largerC.largestD.largely正确答案:C解析:<最高级+in+表示范围的单数名词>26.There have been ______ complaints from local residents about the noise generated on the construction site at Gerber Lane.A.decentB.carelessC.relativeD.frequent正确答案:D解析:frequent频繁的,惯常的27.The members of the hiring committee came to a ______ that Maria Gomez was the best fit for the position.A.terminologyB.consensusC.diagnosisD.formulation正确答案:B解析:consensus合意,一致28.The advent of the Internet has ______ altered the way people do business across the world.A.deliberatelyB.periodicallyC.radicallyD.considerately正确答案:C解析:radically以激进的方式29.Mark Turner earned a promotion due to his ______ strong performance as regional sales manager.A.consistB.consistingC.consistentD.consistently正确答案:D解析:副词位置30.The book series Werewolf Mysteries is extremely popular ______ teenagers and young adults.A.throughoutB.withinC.towardsD.among正确答案:D解析:介词among在……之中(在两者以上的个别对象中)31.Library members ______ to use the computers on the first floor shouldregister at the front desk.A.wantB.wantingC.wantedD.to want正确答案:B解析:形容词位置,表主动的现在分词32.As ______ as Leanne gets back to the office, she will start work on the new advertising project.A.muchB.soonC.farD.many正确答案:B解析:时间连词as soon as(一……就……)33.______ interior decor is one of the most important features that customers look for in a cafe or tea room.A.AttractB.AttractingC.AttractiveD.Attraction正确答案:C解析:形容词位置,形容词优先于分词34.The members of the social committee ______ once a month to discuss upcoming activities and events.A.meetB.meetsC.is meetingD.meeting正确答案:A解析:动词位置,数值一致35.Because the project involves so many different individuals and organizations, the ______ of it will be particularly challenging.A.coordinateB.coordinatorC.coordinatedD.coordination正确答案:D解析:名词位置,coordination(协调)36.It is recommended that any woman over the age of 50 annually ______ the Breast Cancer Screening Center.A.visitB.visitsC.visitedD.visiting正确答案:A解析:<表示“要求,建议,主张”的动词+that+主语+动词原形>37.Store managers employ mystery shoppers to determine what the service encounter feels like from the customer’s ______.A.intentionB.complexionC.perspectiveD.potential正确答案:C解析:perspective观点,视角38.The university’s new research ______ houses a large amount of modern equipment, including high-powered lasers and telescopes.A.facilityB.facilitatesC.facilitatingD.facilitated正确答案:A解析:名词位置,数值一致39.CSP Solutions offers an ______ of financial services to meet the needs of small and medium-sized businesses.A.outputB.arrayC.estimateD.opinion正确答案:B解析:array陈列,大批(an array of一批,大量)40.The council ______ a monument in Civic Square, honoring the soldiers who fought and died in World War II.A.extractedB.locatedC.commissionedD.persuaded正确答案:C解析:commission委任,委托短文填空Attn: All Staff It gives me great pleasure to announce that Leon Fleming (81) the new director of sales and marketing. The hiring committee came to this decision after carefully examining the pool of candidates. It was agreed that Leon is the candidate with the best credentials. Firstly, he is in his 10th year at the company, and before that he worked in the industry for five years. But it is not just Leon’s (82) that made him stand out. He has also worked hard to boost his qualifications, completing a part-time MBA program last year. As director of sales and marketing, Leon will be responsible for a staff of more than 30 employees. He firmly believes his main goal (83) our market share both domestically and overseas will be realized in the near future. I am also fully confident he will achieve this aim. I hope you will all join me in congratulating Leon on this achievement.41.A.is appointingB.is being appointedC.has appointedD.has been appointed正确答案:D解析:时态,语态42.A.enthusiasmB.personalityC.experienceD.ambition正确答案:C解析:experience经验,经历43.A.improvesB.to improveC.improvedD.is improving正确答案:B解析:不定代词to的形容词性用法From: Jenny McCabeTo: Ros SpearsSubject: FundraiserDear Ros, It was lovely to see you at the Local Business Awards last week. During our conversation there, you expressed interest in (84) me organize a fundraising event for the Wildlife Protection Association. I would like to take you up on your kind offer. Having spoken with the members of the Wildlife Protection Association board, I have some interesting ideas for the event. However, (85) put these ideas into practice, we will need to start working on the plans right away. Are you available to meet within the next couple of days to discuss this matter? Please let me know what time suits you. Thanks once again for your assistance. I am sure that together we can help make this fundraiser (86) Regards, Jenny44.A.helpB.helpedC.helpingD.to help正确答案:C解析:起到介词宾语作用的动名词45.A.in order toB.so thatC.in line withD.prior to正确答案:A解析:表目的的in order to do(为了……)46.A.succeedsB.successC.successfulD.successfully正确答案:C解析:形容词位置September 14, 2010Lincoln District Court12 Johnson LaneCentral City, Missouri 63112Dear Ms. Ortega, The Lincoln District Court summons you for jury duty from the week beginning October 27. In accordance with state laws, your name has been chosen (87) random from a list of citizens. Serving on a jury is an important civic responsibility that all citizens may be asked to take part in. If for any reason you are (88) at this time, you must apply for a formal waiver. Valid reasons for waivers may include unavoidable work responsibilities, health problems, or essential caregiving work. Citizens who wish to apply for a waiver must get the appropriate documentation signed by their employer, medical practitioner, or social worker. The required (89) have been enclosed with this letter. Citizens who fail to show up for jury duty and have not received an official waiver will receive a $50 fine. Regards, Christine Turner Jury Officer —Lincoln District Court47.A.inB.byC.onD.at正确答案:D解析:at random随机的48.A.ungratefulB.unwillingC.unavailableD.unworthy正确答案:C解析:unavailable难以获得的;不能利用的49.A.entriesB.formsC.referencesD.specifications正确答案:B解析:form形式,表格From: Helen Pierce, Dallas EngineeringTo: Yolanda Spencer, Opto TechnologySubject: MeetingDear Yolanda, I am writing to let you know I (90) at the Crowne Hotel on Fort Street. I arrived on Monday night tc attend an industry conference. I will head back to Dallas on Saturday morning, so I hope we can meet up sometime on Friday for coffee in the city. Do you think this will be possible? Please (91) to this email to let me know about your availability. If we’re unable to meet this time, then hopefully we can get together at next week’s tradeshow in Pasadena. To be honest, my motive for meeting with you is to lure you to Dallas Engineering. I would love the opportunity to speak to you in person about the new (92) in our executive team. You are just the sort of talented and hard-working professional that we are looking for, and I’m sure you can do well at our firm. During our meeting, I hope I can convince you to consider moving to Dallas! Cheers, Helen Pierce Dallas Engineering50.A.stayB.stayedC.am stayingD.have stayed正确答案:C解析:时态51.A.answerB.postC.submitD.respond正确答案:D解析:respond应答(respond to对…一作出反应)52.A.recruitB.openingC.resourceD.conflict正确答案:B解析:opening空缺的职位阅读理解Grand Opening! Jessica’s is a recently opened exciting new addition to the ranks of the Crosstown Mall. We’ve got the elegant accessories you need to dress up any old outfit. From mother-of-pearl necklaces to sterling silver rings and bracelets, there’s no look or style we can’t create. And, for you morns out there, we even have a children’s video corner so your kids can have some fun while you shop! Come down to Crosstown and see us today. We’re located on the second floor of the east wing, between Stevenson’s Shoes and the stationery shop. To sweeten the deal, bring in this page and we’ll knock 10% off your first purchase!53.What is NOT mentioned about Jessica’s?A.It carries children’s games.B.It sells a variety of jewelry.C.It started its business lately.D.It is next door to an office supply store.正确答案:A54.How can customers receive a discount?A.By bringing in their childrenB.By showing an advertisementC.By purchasing multiple itemsD.By attending the grand opening正确答案:BAttention: Registered participants of the Davidson Woodworking Workshop On September 2, a notice was sent out to everyone who had registered stating that the date of the workshop had been changed from Saturday, October 5 to Saturday, September 28—one week earlier. Unfortunately, this information is incorrect; the date has actually been pushed back one week. None of the other event details have been altered. The workshop will still be held on the Davidson Community College campus from 9:00 am to 5:00 pm, with a one-hour break for lunch. We apologize for this inconvenience and hope to see you there.55.What is the purpose of the notice?A.To encourage registrationB.To correct a mistakeC.To announce a workshop venueD.To provide event details正确答案:B56.When will the workshop be held?A.September 2B.September 28C.October 5D.October 12正确答案:DAfter twenty years of excellent service, but an unfortunate decline in revenues during the last two, Kat’s Restaurant is undergoing a change of ownership. The founder and original owner, Katrina Adams, has plans to open a new upscale Italian restaurant, in contrast to the low-budget cuisine Kat’s has become famous for. Taking her place is restaurateur Patrick Abbas, owner of five other restaurants in the Hyde Park area. He has indicated that the menu will remain largely the same, though he is sketching out plans to fit more tables into the somewhat limited space of the restaurant. Kat’s will close its doors December 1 and will reopen as Pat’s Place sometime in February.57.What aspect of the restaurant is being changed?A.Its staffB.Its cuisineC.Its locationD.Its name正确答案:D58.What is suggested about Mr. Abbas?A.He runs other Italian restaurants.B.He is a professionally trained chef.C.He wants to increase seating capacity.D.He is a newcomer to the restaurant business.正确答案:CDear Mr. Simon, I am writing to you after discovering your entertaining and informative travel website, Around the World. I am not ashamed to admit that I spent a solid two hours perusing your articles on foreign customs and amazing cultural sites, even though I originally stumbled onto it while doing a search for study abroad programs. I feel you’ve done a wonderful job creating content that appeals to a wide variety of people eager to discover new things about the world. Congratulations! Iam a writer by profession, and I recently published my debut work, entitled Lost in France. The story deals with an early travel experience I had in the country and how my memories of it changed when I returned later as an adult. I noticed that your site features several book reviews, and I was wondering if you would consider posting your impressions of my work. I am not looking for a guarantee of praise; your honest opinion is all I ask. If this is a proposition that interests you, I would be happy to send you a copy at my own expense. I only ask that you contact my publisher prior to posting a review so that they may anticipate the publicity. Thanks for your consideration, - Kimi Matsuri59.Why is Ms. Matsuri writing to Mr. Simon?A.To submit a story for publicationB.To congratulate him on the launch of his websiteC.To request a review of her bookD.To complain about an unfair review正确答案:C60.What subject was Ms. Matsuri initially researching?A.Cultural sites around the worldB.Travel book reviewsC.Customs of foreign countriesD.International learning opportunities正确答案:D61.What is indicated about Lost in France?A.It is a work of nonfiction.B.It is a sequel to Around the World.C.It is Mr. Simon’s first novel.D.It is critically acclaimed.正确答案:A62.What does Ms. Matsuri offer to do?A.Ship the bookB.Contact the publisherC.Review the websiteD.Travel with Mr. Simon正确答案:AOfficials at the McNeil Museum of International Art are announcing that themuseum’s March exhibition is going to be something special. It’s all thanks to Mr. Antonio Garcia, one of the museum’s most active members and a man who has donated generously during fundralsers in the past. His most recent contribution, however, has come not in the form of money but rather art itself. For most of his adult life, Mr. Garcia, a native of Colombia, has been collecting both traditional and contemporary works from around South America, including several hundred indigenous weavings from Bolivia. Now, he has gifted a significant portion of these, along with pieces from Peru and Ecuador, to the McNeil. The March exhibition will feature these new acquisitions. Additionally, Tom McClosky, the museum’s general curator, has organized a unique experience for museum members. On February 26, a guided tour will be provided by Mr. Garcia, who will share details about the artwork as well as personal stories of how he acquired it. The exhibition, titled “Andean Artscapes,” is scheduled to open to the general public on March 2.63.What is said about Antonio Garcia?A.He is the museum’s general curator.B.He recently made a large monetary donation.C.He has presented the museum with his collection.D.He is the newest member of the museum.正确答案:C64.Which country’s artwork will NOT be included in the March exhibition?A.PeruB.ColombiaC.EcuadorD.Bolivia正确答案:B65.What is mentioned about museum members?A.They must renew their membership each month,B.They will see the exhibition before the general public.C.They are expected to donate money in return for a tour.D.They will visit the museum together on March 2.正确答案:BFrom: Sue YiTo: Carlton CheeversSubject: Inspection, D4 toaster Carlton, I’m doing a product testing on the ToastMaster D4 Bread Toaster this week, and I’ve encountered a unit that gives me cause for concern. The product number is D4-230943-F, and I’ve found that when all four toaster slots are in use, the unit overheats to dangerous levels. Clearly, there is a problem with the temperature controlapparatus, because according to the product specifications, the maximum heat yield should be well within safety limits. I see here that an inspector from your department, Elisa Chow, signed off on this unit when it came through her station last week. I’d like for you to speak with Ms. Chow and find out how this defect managed to make it past her scrutiny. I’ve already talked to Marshall Lander, the product designer, and he feels this is probably an isolated incident. But we need to get to the root of the problem before we send these toasters out into the market. - Sue 66.What is the purpose of the email?A.To request that a product’s sales be inspectedB.To suggest a change to the inspection processC.To report a problem with a productD.To inquire about the specifications for a product正确答案:C67.Who most likely is in charge of the inspection team?A.Sue YiB.Marshall LanderC.Elisa ChowD.Carlton Cheevers正确答案:D68.What is suggested about the ToastMaster D4 Bread Toaster?A.Its use has resulted in several injuries to customers.B.It was revealed to have commonly known defects.C.It was not designed to toast four slices at once.D.It is not yet available in stores.正确答案:DAttention, employees. As your supervisors should have informed you by now, the building’s electrical system will be undergoing renovations this weekend. The main purpose of the work is to replace all emergency circuit breakers with more modern versions to ensure the safety of all workers. In addition, the system will be upgraded to accommodate the increase in power necessitated by the opening of our new wing of offices next month. Supplementary electrical outlets will also be added in equipment rooms and near all high-use devices, such as printers and copiers. Due to the nature of the work, power to the building will need to be switched off and on several times over the weekend. This could potentially damage any equipment connected to the power grid, so we are asking everyone to unplug all devices, from pencil sharpeners to computer monitors, before they leave for the day. Additionally, therewill be no overtime work performed at our offices this weekend. Anyone who has applied for and received permission for overtime hours should carry out their work at home. Please contact Ms. Suzuki in the tech department with any questions. Thank you.69.What changes are NOT involved in the scheduled work?A.The installation of new power outletsB.Raising the capacity of the electrical systemC.The replacement of old emergency circuit breakersD.Redesigning the offices of the tech department正确答案:D70.What is expected to happen next month?A.Updated office equipment will be purchased.B.An electrical system will be repaired.C.Electrical power will be turned off.D.The company’s workspace will be expanded.正确答案:D71.What are employees instructed to do today?A.Visit the tech departmentB.Apply for overtime permissionC.Disconnect electrical devicesD.Turn their monitors off and on again正确答案:CDear Ms. Amanda Parker, Thank you for your interest in attending the 63rd annual Southeast Regional Salesperson Conference, to be held July 12 through July 14 in the ballroom of the Inter-Regency Hotel. Enclosed is a registration form for you to fill out and send back to reserve a seat. Please note that the registration cost has increased to $150 from last year’s fee of $120. The change is in response to a sharp rise in registrations last year. If we see the same percentage of increase this year, we will not have enough space to accommodate everyone, so it was felt that the higher fee would keep the number of registrants within reason. The deadline for registration is June 15. Please have your form postmarked by that date to participate in regular registration. After June 15, an additional fee of $40 will be applied for expedited registration. And, as always, all registration fees include a non-refundable $50 deposit to protect against cancellation. By affixing your signature to the registration form, you agree to abide by these and all other terms governing the Southeast Regional Salesperson Conference, which can be found on the back of theform. Sincerely yours, Maxine Stinson, Conference Organizer 72.Why did Ms. Stinson send this letter?A.To notify conference participants of a recent fee increaseB.To detail the policies pertaining to conference attendanceC.To report a problem with Ms. Parker’s registration statusD.To encourage conference members to register early正确答案:B73.What is indicated about the conference?A.It has been held continuously for decades.B.Its location is different from last year’s.C.It will have more sessions this year.D.Its registration deadlines are flexible.正确答案:A74.What happens after June 15?A.Registrants are given special status.B.The cost of registration goes up.C.Registrations will no longer be accepted.D.A different registration form will be used.正确答案:B75.How much will participants lose if they cancel their registration?A.$40B.$50C.$120D.$150正确答案:BIt has been just over a year now since the city of Talbot partnered with local firm Construction Waste Processing (CWP) to undertake a massive cleanup project in the warehouse district and other underutilized urban areas. CWP is nationally recognized as a leader in the field of industrial waste recycling, having developed and patented a method of reprocessing concrete, steel, synthetics, and other materials that result from building demolition. Still, at the one-year mark the Talbot City Council called in an independent agency, Waste Management Watchdogs (WMW), to analyze CWP’s progress thus far. The results were released yesterday, and CWP earned a phenomenal score of 29 out of 30. Specifically, WMW judged their cleanup of demolishedstructures in the warehouse district as “excellent.”This comes as good news for the city council, for it means the unprecedented investment of $45 million in the cleanup was wisely spent. CWP will be on the job for another two years, after which time a designer will be chosen to re-imagine and revitalize the warehouse district and downtown Talbot. Talks with several firms have already begun, most notably with the celebrated Canadian organization Celsus Urban Planners. Regardless of the outcome, however, residents of Talbot can be proud that homegrown Construction Waste Processing has performed so admirably, helping the environment at the same time as helping to put Talbot on the industrial services map.76.What is the article mainly about?A.Plans to revitalize a downtown areaB.A method for reusing industrial wasteC.Problems with the Talbot city budgetD.The progress of a recycling project正确答案:D77.What is happening in the warehouse district?A.Modern designs are being experimented with.B.A new company is opening a factory.C.Waste from other projects is being dumped.D.Old buildings are being torn down.正确答案:D78.Which organization was responsible for rating the work completed?A.Celsus Urban PlannersB.Construction Waste ProcessingC.The Talbot City CouncilD.Waste Management Watchdogs正确答案:D79.The word “unprecedented” in paragraph 3, line 2, is closest in meaning to A.moderateB.approvedC.unfortunateD.exceptional正确答案:D80.What is planned to occur in two years?A.Urban areas will be renovated.B.CWP’s contract will be renewed.C.A design firm’s work will be assessed.D.Urban planners will solicit public opinions.正确答案:AFrom: Wendy ThackerayTo: Customer Service Department, One Star TelecomSubject: My May phone statementCustomer Service: I signed up for a cell phone plan with One Star this past March. According to the contract I signed, I was to be charged a base fee of $59 per month for unlimited Internet use and up to 600 minutes of call time. Anything over 600 minutes would be billed at a rate of 20 cents per minute. I chose this plan because I knew I would never surpass the regular minutes. However, in my May phone statement, it clearly shows that I only logged 479 minutes of call time during the month. Yet, the total in the “amount owed”column, before taxes, is listed as $69. Can you explain this discrepancy? I have heard horror stories from friends who receive service from other phone companies about sudden and unannounced rate hikes. I chose One Star specifically because I was led to believe this would never happen. I hope I wasn’t mistaken in that assumption. - Wendy Thackeray From: John Ming, Customer Service Representative To: Wendy Thackeray Subject: Re: My May phone statement Dear Ms. Thackeray, Let me begin by thanking you for bringing this issue to our attention. As you may have guessed, there was an error in the processing of your bill for the month of May. One Star is deeply sorry for the inconvenience, and I assure you that the overcharged amount will be credited to your account as of the June billing cycle. You are correct in believing that One Star will never increase your monthly rate without your knowledge—this was simply the result of a computer glitch. Please allow us to make this up to you and restore your faith in our company. To demonstrate our sincerity, we will immediately raise your monthly call time limit to 700 minutes and throw in unlimited texting. All other details of your plan will remain the same, including the low monthly cost of $59. Also, I recommend that you visit our website and click on the “Customer Survey” button at the top right corner. This will give you a means to provide more details about your experience and help us better serve you and our other customers in the future. Thank you for your understanding and for choosing One Star Telecom. John Ming, Customer Service Representative 81.What is the purpose of the first email?A.To complain about a defective phoneB.To request a review of informationC.To ask for an account to be modifiedD.To terminate a phone contract正确答案:B。

托业(阅读)模拟试卷30(题后含答案及解析)

托业(阅读)模拟试卷30(题后含答案及解析)

托业(阅读)模拟试卷30(题后含答案及解析) 题型有:1. 完成句子 2. 短文填空 3. 阅读理解完成句子1.Business can be classified according to different ______, such as value, number of employees, ownerships, etc.A.criterionB.criteriaC.criticismD.critiques正确答案:B解析:这句话的意思是说,生意可以按照不同的标准分类,如价值,员工人数,所有权等。

(A) criterion(判断的)标准,是单数形式;(B)criteria是criterion 的复数形式,是不规则的变化;(C)criticism批评、评论;(D)critiques名词的复数形式,是“批评、批评法”的意思。

2.The sole owner, often aided by one or two ______, operates a small shop that frequently caters to a group of regular customers.A.employersB.employeesC.customersD.managers正确答案:B解析:这句话的意思是说,个人企业的所有人,总是雇佣一两个人,开一家小商店来迎合一些固定顾客的需求。

(A)employer雇主;(B)employee雇员;(C)customer顾客;(D)manager管理者。

3.Regardless of the percentage of the business they own, all partners have authority to act and to make______ decisions as owners of the business.A.bindingB.responsibleC.reliableD.dependable正确答案:A解析:这句话的意思是说,不管他们拥有生意的百分比是多少,所有的参与者都有权对他们自己那部分作出附有约束力的决定。

托业(阅读)模拟试卷45(题后含答案及解析)

托业(阅读)模拟试卷45(题后含答案及解析)

托业(阅读)模拟试卷45(题后含答案及解析) 题型有:1. 完成句子 2. 短文填空 3. 阅读理解完成句子1.Every Saturday afternoon in a parking lot on Cedar Street, collectors come to ______off their prized vintage cars.A.showB.turnC.holdD.watch正确答案:A解析:由于是为了展示自己珍藏的老爷车而进行的集会,所以(A)show是正确答案。

show off是“炫耀,夸耀”的意思。

2.Everybody______has a company-issued cellular phone should e-mail their name and phone number to Cassandra Miller by 5 P.M. tomorrow.A.whoB.whomC.whichD.where正确答案:A解析:这句话的主语是Everybody,动词是should e-mail。

空格中需要填人修饰先行词everybody的关系代词,先行词是人并且后面出现了动词has,所以应该选择表示主格的关系代词who。

3.The most______complaint among the staff is not having enough opportunities for professional development.A.commonB.commonlyC.is commonD.is commonly正确答案:A解析:空格部分需要选择既能被最高级副词(the)most修饰又能修饰后面的名词complaint的词汇,所以(A)common为正确答案。

4.Because this month has been unusually ______, my team and I have been able to take the time to clean out file cabinets and archive old project materials.A.slowB.busyC.lateD.active正确答案:A解析:由于整理文件需要时间所以空格中应填人表示“(速度、进程)不快”,“缓慢”的选项(A)slow。

托业(阅读)模拟试卷4(题后含答案及解析)

托业(阅读)模拟试卷4(题后含答案及解析)

托业(阅读)模拟试卷4(题后含答案及解析) 题型有:1. 完成句子 2. 短文填空 3. 阅读理解完成句子1.A ______firm will help us find software.A.consultationB.consultantC.consultingD.consult正确答案:C解析:Consulting firm is a business term. Choice (A) is a noun referring to a thing. Choice (B) is a noun referring to a person. Choice (D) a verb.2.______ Mr. Jeffries to get the job done.A.Count onB.Count fromC.Count inD.Count up正确答案:A解析:Count on means to depend on. Choice (B) means to start counting at a particular point. Choice (C) means to include. Choice (D) means to total.3.Ms. Nyguen had submitted her résumébefore she ______the position was filled.A.will knowB.knowsC.has knownD.knew正确答案:D解析:A past action that happens after a previous past action is in the simple past tense. Choice (A) is the future tense. Choice (B) is the present tense. Choice (C) is the present perfect.4.If Mr. Donna were looking for a permanent job, our recruiter ______ help.A.mayB.willC.canD.could正确答案:D解析:An unreal condition in the present tense may use could in the result clause. Choices (A), (B), and (C) are not possible modals for an unreal condition.5.The purchaser wanted the equipment ______ by Monday morning.A.deliveredB.deliveringC.will be deliveredD.must be delivered正确答案:A解析:Someone else will deliver the equipment, so the past participle is used. Choice (B) is the present participle. Choices (C) and (D) cannot follow want.6.The company’s quarterly earnings were up;, the ______officers felt satisfied.A.neverthelessB.thereforeC.howeverD.for this purpose正确答案:B解析:Therefore indicates a cause-and-effect relationship. Choices (A) and (C) indicate an unexpected result. Choice (D) indicates purpose.7.______ substantial layoffs, costs were reduced.A.WhenB.Because ofC.AlthoughD.Since正确答案:B解析:Because of indicates a cause-and- effect relationship and can be followed by a noun phrase. Choice (A) must be followed by a subject and verb. Choice (C) indicates an unexpected result and must be followed by a subject and verb. Choice (D) must be followed by a subject and verb when indicating a cause-and-effect relationship.8.Consumer confidence fell______ April.A.nextB.onC.inD.the正确答案:C解析:In is used to indicate time during a month. Choice (A) may be used with month but indicates the immediate future. Choice (B) is used with dates. Choice (D), the, is not used with names of months.9.Price quotes ______.A.have daily been announcedB.have been announced dailyC.daily have been announcedD.have been daily announced正确答案:B解析:Adverbs of definite frequency may appear at the end of a sentence. Choices (A), (C), and (D) are not appropriate positions for definite frequency adverbs.10.The administration allows Thailand ______Indonesia trade benefits.A.butB.norC.and soD.and正确答案:D解析:And joins items equally. Choice (A) indicates a contrast between items. Choice (B) is usually paired with neither. Choice (C) indicates sequence or cause and effect.11.The talks will take place ______ Brussels.A.atB.theC.inD.to正确答案:C解析:In is used to indicate location within cities. Choice (A) is used with a specific time. Choice (B) is an article, not a preposition. Choice (D) indicates direction toward.12.Many workers can’t use computers; ______ training is required.A.on the wholeB.besidesC.consequentlyD.for example正确答案:C解析:Consequently’indicates cause and effect. Choice (A) summarizes previous points. Choice (B) adds additional information. Choice (D) indicates an example.13.Management let the employees ______at two o’clock.A.leaveB.leftC.was leavingD.was left正确答案:A解析:The employees are the ones who are leaving, so the simple form of the verb is used. Choice (B) is the past tense. Choice (C) is the past continuous. Choice (D) is the past tense in the passive form.14.If our candidates ______elected, we’ll have a strong board.A.areB.wereC.have beenD.will be正确答案:A解析:Present or future possible conditions use present tense in the if- clause. Choice (B) is required in unreal conditions with be. Choice (C) is the present perfect. Choice (D) is the future.15.It’s important that the clients ______interested.A.are seemingB.will seemC.is seemingD.seem正确答案:D解析:The clients are the ones who seem interested, so the simple form of the verb is used. Choices (A) and (C) are participles, which are rarely used with stative verbs. Choice (B) is the future tense.16.Find ______the details and write a report.A.upB.aboutC.outD.around正确答案:C解析:Find out is a two-word verb meaning to uncover information. Choices (A),(B), and (D) do not complete the verb.17.The meeting ______going on since eight o’clock this morning.A.has beenB.wasC.isD.will be正确答案:A解析:An action that starts in the past and continues to the present uses the present perfect tense. Choice (B) is the past tense. Choice (C) is the present. Choice (D) is the future.18.The consultant ______his business if he had advertised.A.doubledB.will doubleC.would doubleD.could have doubled正确答案:D解析:A past unreal condition that uses the past perfect in the if-clause uses could have or would have and the past participle in the result clause. Choice (A) is the past tense. Choice (B) is the future. Choice (C) is the present conditional.19.Mr. Dalla would like the invoices ______directly to Milan.A.faxB.faxingC.be faxedD.faxed正确答案:D解析:Someone else will fax the invoices, so the past participle is used. Choice (A) is the simple form. Choice (B) is the present participle. Choice (C) is passive.20.The members would resign if they ______asked to do so.A.areB.wereC.will beD.would be正确答案:B解析:Were is the form of be required for the if-clause of an unreal condition. Choice (A) is the present tense. Choice (C) is the future. Choice (D) is conditional.21.Ambition, talent, ______desire are ingredients for success.A.orB.neverC.yetD.and正确答案:D解析:And joins items equally. Choice (A) indicates a choice among items. Choices (B) and (C) are not coordinating conjunctions.22.Only five years ago, there ______a shortage of computer specialists.A.wasB.wereC.has beenD.have been正确答案:A解析:The simple past tense is required. Choice (B) is past tense but does not agree with the singular subject shortage. Choice (C) is present perfect. Choice (D) is present perfect and does not agree with shortage.23.The board reported that more funds ______for training.A.was givenB.could have givenC.should be givenD.is given正确答案:C解析:Should be given indicates an intention or a recommendation. Choice (A) is past tense passive and does not agree with funds. Choice (B) requires an active subject. Choice (D) is given is present tense passive and does not agree with funds.24.Some employees get their paychecks automatically ______in their bank accounts.A.depositedB.depositingC.are depositedD.deposit正确答案:A解析:Someone else will deposit the paychecks, so the past participle is used. Choice (B) is the present participle. Choice (C) is the present tense passive. Choice (D) is the simple form.25.Costs should be cut; ______, the number of staff positions will be reduced.A.howeverB.thereforeC.meanwhileD.but正确答案:B解析:Therefore indicates a cause and effect or a result. Choice (A) indicates an unexpected result. Choice (C) indicates simultaneous occurrence. Choice (D) indicates a contrast.26.Office branches are located______ the metropolitan area.A.onB.atC.aboutD.throughout正确答案:D解析:Throughout is used to mean many different locations within a city or another area. Choices (A) and(B) may be used with street locations but not with cities. Choice (C) means concerning.27.The company offers a ______plan for its workers.A.retirementB.retiringC.retireD.retired正确答案:A解析:Retirement plan is a business term. Choice (B) is the present participle. Choice (C) is the simple form. Choice (D) is the past participle.28.______ Mr. Hague finished the job interview, he felt relieved.A.WhileB.Because ofC.DuringD.After正确答案:D解析:After indicates a time sequence for events. Choice (A) indicates simultaneous action. Choice (B) indicates cause and effect and is not followed by a subject and verb. Choice (C) indicates simultaneous action but is not followed by a subject and verb.29.Ms. Lopez has ______learned to create zip files.A.yetB.everC.alreadyD.certain正确答案:C解析:Already is an adverb that indicates an action that has taken place sooner than expected. Choice (A) indicates the end of a time limit. Choice (B) means at any point in time. Choice (D) is an adjective.30.The paychecks will be delivered ______they arrive from the accounting department.A.beforeB.soonC.whenD.during正确答案:C解析:When indicates a time sequence, especially where one action is influenced by another. Choice (A) is not logical. Choice (B) is not the correct form as soon as. Choice (D) cannot be followed by a subject and a verb.31.The report outlines the products for the first quarter ______ the year.A.toB.atC.fromD.of正确答案:D解析:Of is used to relate portions of time to the whole. Choice (A) indicates direction toward. Choice (B) is used with specific times of day. Choice (C) indicates source or origin.32.The benefits package is impressive; ______, the director promotes only from within the company.A.for exampleB.whenC.despiteD.nevertheless正确答案:A解析:For example indicates a specific instance of a more general statement. Choice (B) indicates time sequence. Choices (C) and (D) indicate unexpected results.33.This company attempts to make its employees ______like family.A.feelingB.feelsC.feltD.feel正确答案:D解析:The employees are the ones who will feel like family, so the simple form of the verb is used. Choice (A) is the present participle. Choice (B) is the present tense singular. Choice (C) is the past tense.34.If the bills ______in five days, the company will seek damages.A.weren’t paidB.won’t have paidC.aren’t paidD.don’t pay正确答案:C解析:Present real conditions may use present tense passive in the if-clause. Choice (A) is past tense passive. Choice (B) is future perfect passive. Choice (D) is present tense but is an active form.35.The bank ______another branch in Houston within the next year.A.openedB.will be openingC.have openedD.would open正确答案:B解析:Will be opening indicates a future time consistent with within the next year. Choice (A) is past tense. Choice (C) is’, present perfect. Choice (D) is conditional.36.Could you have the assistant ______my office before he leaves today?A.stop offB.stop forC.stop toD.stop by正确答案:D解析:Stop by is a two-word verb that means to visit someone briefly. Choice (A) means to interrupt a trip briefly. Choice (B) means to stop for a specific purpose (followed by a noun, as in stop for gas). Choice (C) means stop in order to complete an action.37.The ______was settled on Saturday.A.disputeB.disputingC.disputableD.disputant正确答案:A解析:Dispute means an argument or a disagreement. Choice (B) is the gerund. Choice (C) is an adjective. Choice (D) is a noun referring to a person in a dispute.38.Make sure to use an ______dealer.A.authorityB.authorizationC.authorizedD.authoritarian正确答案:C解析:Authorized is an adjective meaning a dealer who is authorized (to deal with a particular product or service). Choice (A) is a noun referring to a person. Choice (B) is a noun referring to a thing. Choice (D) is an adjective meaning requiring dependence upon authority.39.The printer apologized for______ two names on the program.A.leaving forB.leaving outC.leaving toD.leaving from正确答案:B解析:Leave out is a two-word verb meaning to omit. Choice (A) indicates leaving with a destination in mind. Choice (C) means to bequeath a possession. Choice (D) indicates leaving a location.40.Mr. Fox ______the results tomorrow afternoon.A.will be knowingB.will knowC.will have knownD.is going to be knowing正确答案:B解析:The simple future tense is required. Choices (A) and (D) are the present continuous. Choice (C) is the future perfect.短文填空Office Works 544 Hudson Street Boston, MA 34602 Tel: (617) 555-7664 Fax: (617) 555-7670 May 10, 20__ Mary Briddock Banqueting Director Wynd’s Garden Hotel 219 Center Circle Boston, MA 03299 Dear Ms. Briddock: Our company, Office Works, is seeking a place to host a banquet. We will honor our top employees at this event, which will include dinner followed by speeches and the presentation of awards. At the same time we plan to celebrate an (141) to our company. We recently purchased the Office Supply Store, and we are now the region’s largest seller of office supplies. We expect approximately 100 guests. We would like to have our event on Saturday, August 15th. If no room is available for that date, we could consider (142) it on the following Saturday, August 22ha. Could you please mail me your latest price list, descriptive brochure, and menus? I am interested in learning more about (143) facilities and services. Thank you. Lynn Ishii Events Coordinator 41.A.additionB.additiveC.addendD.addendum正确答案:A解析:An addition is a piece or section added to something. The other choices have a similar meaning but more specific uses. Choice (B) usually refers to chemicals. Choice (C) is a term used in arithmetic. Choice (D) is an addition to a document.42.A.haveB.to haveC.havingD.will have正确答案:C解析:The verb consider is followed by a gerund. Choice (A) is the base form. Choice (B) is the infinitive form. Choice (D) is the future form.43.A.myB.ourC.yourD.their正确答案:C解析:The second-person possessive adjective your refers to the person to whom the letter is addressed; the facilities and services are those of the hotel where that person works. Choices (A) and (B) are the first-person adjectives. Choice (D) is third person.GLOBAL AIRLINES POLICY STATEMENT Baggage Each passenger may carry two items onto the plane: one small suitcase and one personal item. The personal item may be a purse, laptop computer, or briefcase. The items should fit under the seat in front of you or in the overhead bin. These bins fill quickly, so in case of overload, the flight attendant may place your suitcase in the back of the plane. In addition (144) the two carry-on items, each passenger may check two suitcases to be transported in the airplane’s luggage compartment. These suitcases must meet the airline’s size limits, (145) baggage charges apply to oversized and additional pieces. These charges must be paid at the time of check-in. Please (146) a customer service representative if you have questions about this policy.44.A.ofB.toC.withD.from正确答案:B解析:The correct form of the expression is in addition to. Choices (A), (C), and (D) are prepositions that cannot be correctly used with this expression.45.A.ExcessB.ExcelledC.ExceptedD.Executive正确答案:A解析:When passengers have too much, or excess, baggage, they have to pay an extra charge. Choices (B), (C), and (D) look similar to the correct answer but have very different meanings.46.A.contactB.contactsC.to contactD.can contact正确答案:A解析:This is an imperative sentence, advising or telling the reader what to do in case of questions. This type of sentence doesn’t require a subject. The other choices are verb forms that require a subject. Choice (B) is a present tense verb. Choice (C) is the infinitive form. Choice (D) is a modal plus verb.Is dentistry the career for you? Today’s dentists have many more opportunities than they used to. In the past, a dentist’s life was predictable. Most were men who would leave home for a few years to study at a dental school, then return to their hometown, open up an office, and work there their whole life. Most dentists looked forward to this or something similar. Today, dentists are more mobile. Men and women can (147) dental school in different places. Some still return to their hometown to work. Others move on to new cities. Dentists today may specialize in one or more areas. Two common ones are oral surgery and dental public health. Oral surgery focuses (148) treating diseases and problems through operations. Dental public health concentrates on improving the dental health of a community. For example, these dentists (149) to schools to teach children how to brush their teeth. Some dentists work only in their specialty areas while others do both general and specialty work.47.A.will attendB.attendingC.to attendD.attend正确答案:D解析:A modal verb, such as can, must be followed by the base form of a verb Choice (A) is a future form. Choice (B) is a present participle. Choice (C) is an infinitive.48.A.inB.atC.onD.to正确答案:C解析:The verb focus is followed by the preposition on. Choices (A), (B), and (D) are prepositions that cannot follow the verb focus.49.A.goB.wentC.had goneD.would go正确答案:A解析:This sentence describes what public-health dentists normally or habitually do, so the simple present tense is used. Choice (B) is simple past tense. Choice (C) is past perfect tense. Choice (D) is a conditional form.To: frontofficel From: geraldinebennett Re: Doing our part Dear Mr. Cobalt: I’m writing to you to express my concern about Pascal’s reputation as a wasteful company. I’m sure you read the recent letter to the editor in the Sydney Daily News regarding our non-recyclable plastic bottles and caps. Since the letter was printed, (150) have received complaints from hundreds of customers who are threatening to boycott our company if we don’t change our practices. We have also received flyers from (151) manufacturing companies offering to help us become a more environmentally friendly company. ! have looked briefly into some of these options on my own time but would like to ask your permission to do more. I would like (152) 20 hours of my upcoming workweek to this cause in hopes of coming up with a viable plan for Pascal’s future as a company that cares about recycling, I hope you will agree with me that our environment is worth us making an effort. Please respond as soon as possible. Sincerely, Geraldine Bennett, Administrative AssistantPascal’s Pharmaceuticals50.A.weB.he and IC.theyD.your正确答案:A解析:We refers to the writer of the letter and her fellow staff members. Choices (B) and (C) don’t have any meaning in this context. Choice (D) is a possessive adjective so cannot be used as the subject of the sentence.51.A.variesB.varietyC.variousD.variable正确答案:C解析:Various is an adjective that describes manufacturing companies. Choice (A) is a present tense verb. Choice (B) is a noun. Choice (D) is an adjective but has a meaning that doesn’t fit the context.52.A.devoteB.devotingC.to devoteD.devotion正确答案:C解析:Would like is followed by an infinitive verb form. Choice (A) is a present tense verb. Choice (B) is a present participle. Choice (D) is a noun.阅读理解Memorandum To: Juan Gomez From: Maria Johnson, Building Engineer Date: 17 Jan. 20__ Re: Thermostat located in your office It has come to our attention that the thermostat located in your office is frequently being turned off. Please be aware that although this thermostat is located in your office, it actually controls the temperature on the entire second floor. When it is turned off, it affects not only your office, but all the surrounding offices as well. We ask that you not touch the thermostat. The other second floor tenants are complaining about the lack of heat in their offices. If you wish to adjust the temperature in your office at any time, please speak with me or with one of my assistants. We would be happy to help you create an environment that is comfortable for you and your office staff, but please remember that we need to consider the comfort of everyone in the building. Thank you for your cooperation.53.What is the problem?A.Someone keeps turning off the thermostat.B.There is no thermostat on the second floor.C.The other tenants want a thermostat.D.The second floor has enough heat.正确答案:A解析:The problem is that someone in Mr. Gomez’s office keeps turning off the thermostat. Choice (B) is incorrect because there is a thermostat in Mr. Gomez’s office. Choice (C) confuses the other tenants want a thermostat and the other tenants are complaining about the lack of heat. Choice (D) is contradicted by the tenants complaining about the lack of heat.54.When should the thermostat be turned off?A.In the eveningsB.When it gets cold outC.NeverD.When it gets hot正确答案:C解析:We ask that you not touch the thermostat means that it should never be turned off. Choices (A), (B), and (D) are times when people usually adjust their thermostats.55.The word “adjust” in paragraph 2, line 1 is closest in meaning toA.pay forB.changeC.look atD.open正确答案:B解析:Adjust means to alter or change. Choice (A) is confused with the fact that one has to pay for heating, but this message is not about heating bills. Choice (C) is something one does to the temperature in a room, but it doesn’t fit the context. Choice (D) is a word that cannot be used with temperature.56.What should Mr. Gomez do?A.Change officesB.Turn off the heatC.Leave the thermostat aloneD.Complain to the other tenants正确答案:C解析:The purpose of the memo is to ask Mr. Gomez not to touch the thermostat. Choice (A) is something Mr. Gomez might do if he is seeking a mere comfortable temperature, but it isn’t mentioned. Choice (B) is what the memo asks Mr. Gomez not to do. Choice (D) is confused with the other second-floor tenants are complaining.White Shoe Kleen-Kit White shoes are a handsome addition to any summer wardrobe, but they have always been difficult to keep clean.., until now. Wright and Perry, the same company that has been providing you with top quality shoe finishes and other fine shoe care products for years, has developed a solution to the problem of cleaning white shoes. Thorough research and careful testing of trial products have resulted in Kleen-Kit, the fantastic new two-step, two-minute product that will keep YOUR white shoes sparkling white. Our special formula not only cleans your shoes to their whitest, it also protects them from dirt and water and preserves the leather, giving your shoes longer life. If you own a pair of white shoes, or plan to enjoythat extra sparkle that they can add to your wardrobe, this kit is a must. It solves the problem you have always had.., of keeping white shoes white. And, at a price you can afford. Kleen-Kit sells for only $7 each, or $5 with each shoe order. Kleen-Kit is available at most shoe retail outlets and anywhere shoe-care products are sold. Not convinced? Ask your local shoe dealer for a free trial sample or request one from our website. Your shoes will be sparkling white in no time!57.What is this advertisement promoting?A.White shoesB.Shoe cleanerC.Shoe repairD.Company supplies正确答案:B解析:The advertisement is promoting a cleaning kit for white shoes. Choice (A) is what the product keeps clean, ,Choice (C) confuses shoe repair with shoe care. Choice (D) is not mentioned.58.How long does it take to use the kit?A.Two minutesB.Five minutesC.Seven minutesD.Ten minutes正确答案:A解析:The kit is a two-step, two-minute kit. Choice (B) confuses five minutes and five dollars, the price of the kit with a shoe order. Choice (C) confuses seven minutes and seven dollars, the cost of the kit. Choice (D) is not mentioned.59.What problem does the product solve?A.Improving Wright shoe salesB.Finishing firstC.Staying handsomeD.Keeping white shoes white正确答案:D解析:The Kleen-Kit keeps white shoes white. Choice (A) confuses the similar words Wright and white. Choice (B) confuses the similar words finishing and finishes. Choice (C) confuses staying handsome and white shoes are handsome.From: MS, J. Gibbons To: All employees Date: July 17,20__ Subject: Required hours All employees are reminded that the Smlthson Company observes a 35-hour workweek, normally from 9:00 A.M. to 5:00P.M., Monday through Friday, with a one-hour lunch period. In addition, after careful consideration and planning, the Smithson Company has recently Implemented a flex-time policy. This means that Individual employees, with good reason, may establish different work schedules with their supervlsor’s approval. The employee must explain the reasons for requesting flex-time as well as demonstrate that a variation of work hours will not be detrimental in any way to the work of the other people on that employee’s team. Whatever changes may be made to an Individual’s schedule, 35 hours per week is still expected of all full-time employees. Flex-time does not mean that an employee can work any hours he or she chooses at any time. In order to make schedule changes under the flex-time policy, an approval form must be submitted to your supervisor two weeks In advance of the proposed schedule change date. The form must be signed and filed by your supervisor before you can start following a new schedule. Forms are available from the Human Resources Office. Employees who adjust their schedule without completing a form and getting formal supervisor approval are subject to loss of pay, even If they work a full 35 hours In a week.60.What is the topic of the e-mail?A.SchedulesB.HiringC.ObservationsD.Lunches正确答案:A解析:The e-mail discusses work schedules. Choice (B) is not mentioned. Choice (C) confuses the similar words observations and observes. Choice (D) repeats the word lunch, mentioned as part of the daily schedule.61.How many hours a week do the employees work?A.17 hoursB.25 hoursC.35 hoursD.40 hours正确答案:C解析:The company observes a 35-:hour workweek. Choice (A) is the day the e-mail was written. Choice (B) confuses the similar words twenty-five’ and thirty-five. Choice (D) is a national standard workweek in the United States.62.Who must approve any change?A.The employeeB.The employee’s supervisorC.The human resources directorD.The vice-president正确答案:B解析:Individual employees may establish different schedules with their supervisor’s approval. Choice (A) is contradicted by with their supervisor’s approval. Choice (C) may be Ms. Gibbons’s position. Choice (D) is not mentioned.MEMORANDUM To: All employees From: George Hendriks, Chief of Security Date: May 30, 20__ Re: Office Visitors Employees are reminded that a number of our contracts with clients are of a confidential nature. In order to ensure the maintenance of this confidentiality, visitors will not be allowed within the office area unless they are accompanied by a member of the staff. Please let your visitors know that they will be asked to sign in at the reception desk when they enter the building. The receptionist will call your office to admit them and give them a visitor’s pass. You must come to the reception area and escort your visitors to your office. Unescorted visitors will be asked by security to leave the building. Thank you for your cooperation with this matter.63.Who will read this memo?A.ClientsB.Company employeesC.VisitorsD.Security staff正确答案:B解析:The memo is addressed to all employees. Choice (A) confuses clients reading the memo and contracts with clients. Choice (C) is the subject of the memo. Choice (D) is who wrote the memo.64.Why must visitors not be alone?A.They may get lost.B.They have appointments with staff members.C.Company projects are confidential.D.They are special guests.正确答案:C解析:Visitors must not be alone because a number of the company’s contracts with clients are of a confidential nature. Choice (A) is not mentioned. Choice (B) is why visitors would come to the office. Choice (D) associates visitors with , guests.65.What must visitors do when they arrive?A.Call the office they are visitingB.Admit themselves。

托业(阅读)模拟试卷25(题后含答案及解析)

托业(阅读)模拟试卷25(题后含答案及解析)

托业(阅读)模拟试卷25(题后含答案及解析) 题型有:1. 完成句子 2. 短文填空 3. 阅读理解完成句子1.The young man was advised to______a prescription by the doctor in order to get recovered as quickly as possible.A.createB.fillC.informD.make正确答案:B解析:这句话的意思是说,医生建议那个小伙子照方抓药以便尽快康复。

fill 基本义是“装满、填充”,fill a prescription为习惯搭配,表示“根据药方抓药”。

(A)create创造、创立;(C)inform通知、告诉;(D)make制作、使用。

2.Before taking off, the flight crew have to make sure that ______ of the passengers has fastened their seat belt and is sitting in an upright position.A.everyB.allC.muchD.each正确答案:D解析:这句话的意思是说,起飞前,机组人员必须保证每一位乘客都已系好安全带并端坐在座位上。

(A)every每一个、每一位,强调整体,谓语动词用第三人称单数,但其后不接of;(B)all全部,所有,强调整体,谓语动词应为复数;(C)much许多,修饰不可数名词;(D)each每一个、每一位,强调部分,谓语动词为单数形式,可跟介词of。

3.The production line in our factory has gone wrong without any______cause for almost three weeks.A.negligibleB.prerequisiteC.apparentD.contingent正确答案:C解析:这句话的意思是说,没有任何明显的起因,我们工厂的生产线就出了故障,这都已将近三个星期了。

托业(阅读)模拟试卷74(题后含答案及解析)

托业(阅读)模拟试卷74(题后含答案及解析)

托业(阅读)模拟试卷74(题后含答案及解析) 题型有:1. 完成句子 2. 短文填空 3. 阅读理解完成句子1.Keynote speaker Melanie Allen will send the draft of______speech to the events committee on Thursday.A.sheB.hersC.herD.herself正确答案:C解析:名词speech前面应该使用具有定语功能的物主代词,因此正确答案是形容词性物主代词(C)her。

人称代词(A),名词性物主代词(B),反身代词(D)都不能修饰名词。

2.Should customers need additional______ about the features of the company’s appliances, they may call our toll-free number to speak to a technician.A.informB.informsC.informationD.informed正确答案:C解析:前面有形容词additional修饰,因此后面应接名词,故选(C)information(信息)。

形容词不能修饰动词(A)和(B),也不能修饰动词的过去分词形式(D)。

考生需要了解句子“Should customers need…”是省略了条件状语从句“If customers should+动词原形(need)”中的if之后,将主语customers与助动词should倒置得到的。

3.Please fill out the enclosed customer data sheet______we can let you know about our latest offerings and promotions.A.instead ofB.as much asC.so thatD.as well as正确答案:C解析:这句话中的主要成分“Please fill out…sheet”为一完整分句,故应将后面的“——we can let…promotions”看作状语从句。

托业(阅读)模拟试卷80(题后含答案及解析)

托业(阅读)模拟试卷80(题后含答案及解析)

托业(阅读)模拟试卷80(题后含答案及解析) 题型有:1. 完成句子 2. 短文填空 3. 阅读理解完成句子1.After the members of the volleyball team attended the awards ceremony, all of ______went to Daphne’s Grill to celebrate the victory.A.theyB.themC.theirD.theirs正确答案:B解析:可以作of宾语的宾格人称代词(B)和名词性物主代词(D)都有可能是正确选项。

根据“他们所有人都去了Daphne’s Grill”的语意,可以推断宾格人称代词(B)them是正确答案。

使用(D)theirs时,表示“他们的所有人都去了Daphne’s Grill”,句意不通顺。

主格人称代名词(A)和形容词性物主代词(C)都不能单独用作of的宾语。

2.Coeval Museum of History recently opened a new______as part of its summer calendar of events.A.exhibitoryB.exhibitorC.exhibitD.exhibited正确答案:C解析:空格处所填的单词既可以作opened的宾语,同时又可以被形容词new 修饰,故(B)和(C)都有可能是正确答案。

根据句意“作为夏日活动日程的一部分,开设了新的展示会”,可知抽象名词(C)exhibit(展示会)是正确答案。

将人物名词(B)exhibitor(展示者)带入句中会使句意不通,故排除。

形容词(A)和动词的过去分词(D)均不能作宾语。

3.All questions regarding the preparations for the company gala should be directed to either Eula______Jeanne.A.orB.yetC.butD.nor正确答案:A解析:连词either…or…是固定搭配(either A or B:A或者B,二者之一),因此(A)or是正确答案。

托业(阅读)模拟试卷89(题后含答案及解析)

托业(阅读)模拟试卷89(题后含答案及解析)

托业(阅读)模拟试卷89(题后含答案及解析) 题型有:1. 完成句子 2. 短文填空 3. 阅读理解完成句子1.If time permits, guests will______visit the waterfalls on the island or explore the botanical gardens.A.eitherB.sinceC.not onlyD.unless正确答案:A解析:与空格后的or一起构成相关连词either A or B(A和B中任一)结构的(A)either为正确答案。

作为参考,either…or…连接助动词will后的动词短语visit the waterfalls与explore the botanical gardens。

2.During the staff meeting, Ms. Crabbe emphasized the importance of observing office dress codes at______times.A.allB.onlyC.entireD.whole正确答案:A解析:形容词(A)all(全部的)能与空格前的介词at和空格后的名词times连用,构成表“总是”之意的搭配at all times,因此(A)为正确答案。

at all times(总是)可作为惯用语来记忆。

(B)only意为“唯一的”,(C)entire意为“全体的”,(D)whole 意为“全部的”。

3.It is the responsibility of agents at the airline’s check-in counters to have baggage tags on hand for travelers who ______them.A.requestingB.requestC.requestsD.to request正确答案:B解析:关系代词作主语的定语从句“who______them”中缺少谓语动词,故动词(B)和(C)为备选答案。

托业(阅读)模拟试卷27(题后含答案及解析)

托业(阅读)模拟试卷27(题后含答案及解析)

托业(阅读)模拟试卷27(题后含答案及解析) 题型有:1. 完成句子 2. 短文填空 3. 阅读理解完成句子1.Any new products are given ______market testing so that the restaurant will have a reasonable idea of its potential before it is added to the menu.A.sternB.hardC.rigorousD.severe正确答案:C解析:这句话的意思是说,所有的新产品都要经过严格的市场测试,以便饭店在把它们添入菜单前对其潜力有一个合理的认识。

(A)stern严肃的,多形容人;(B)hard困难的;(C)rigorous不但有“严格的”意思,还有“严密的、精密的”意思;(D)severe严重的、严肃的。

2.The manager ______ the work of others to accomplish goals that might not be achievable by an individual.A.cooperatesB.arrangesC.matchesD.coordinates正确答案:D解析:这句话的意思是说,管理者协调其他人的工作,为的是完成那些一个人不能完成的任务。

(A)cooperate合作;(B)arrange安排,强调对事物在顺序或关系上调整;(C)match使一致、配合;(D)coordinate协调,侧重“实现良好的结果”。

3.Over a third of the population was estimated to have no ______ to health service.A.admissionB.accessC.entranceD.accept正确答案:B解析:这句话的意思是说,估计有超过三分之一的人口没有权利享受医疗服务。

托业(阅读)模拟试卷50(题后含答案及解析)_0

托业(阅读)模拟试卷50(题后含答案及解析)_0

托业(阅读)模拟试卷50(题后含答案及解析) 题型有:1. 完成句子 2. 短文填空 3. 阅读理解完成句子1.We invest funds on_______of over 120 clients including pension funds, insurance companies, and government entities, as well as financial institutions and charities.A.accountB.behalfC.instructD.order正确答案:B解析:投资公司是代客户投资的公司。

on behalf of是“代表……”的意思。

如果C选项是instruction,变成on the instruction of,意为“在……的指示下”,则可以被选为正确答案。

2.The results of this marketing survey are_______with previous findings.A.collectedB.satisfiedC.consistentD.assembled正确答案:C解析:请把“be+形容词+介词”当作惯用搭配来记忆。

be consistent with意为“符合的……”或“与……一致的”。

B选项也和with搭配使用,但词义上不合适。

3.The purpose of my coming here is______business with you.A.doB.to doC.doingD.done正确答案:B解析:这句话的意思是说,我这次来是想和你们开展业务往来。

不定式可以用来表示尚未发生、一次性的和具体的动作,所以正确项为(B)。

4.If a better material______, the cost of the product would have been decreased.A.had been usedB.had been usingC.being usedD.using正确答案:A解析:if引导的条件从句如果对过去事实进行虚拟,从句时态是if…were/did…,而主句谓语动词用情态动词过去式加上动词原形。

托业(阅读)模拟试卷8(题后含答案及解析)

托业(阅读)模拟试卷8(题后含答案及解析)

托业(阅读)模拟试卷8(题后含答案及解析) 题型有:1. 完成句子 2. 短文填空 3. 阅读理解完成句子1.Because all production was_______ out to other firms, we can concentrate more on making business strategies, which is more cost-effective.A.contractedB.convertedC.compiledD.confirmed正确答案:A解析:本题必须理解整个句子才能解题,具有相当的难度。

句中说,因为生产方面已经和其他公司“签订了合同”,所以精力应更多地集中于经营策略方面。

签约(contract)显然是“对外”进行的。

2.As you all know, because of our recent economic difficulties, we should move the office equipment_______.A.oursB.ourselvesC.hisD.himself正确答案:B解析:空格前面是个“S+V+O”形式的完整句子,所以不需要其他成分了。

这种情况就是反身代词表示强调的使用方法。

就如同一个副词。

起强调句意的作用。

这里只要选择和动作主体we一致的词语即可。

3.We cannot process your order_______we have received the deposit mentioned in our mutually agreed contract.A.whileB.duringC.untilD.if正确答案:C解析:空格内应填入连词,此时需要通过分析词语来选择答案。

until既可以做介词,又可以做连词。

句意是必须在收到订金之后才能进行处理,所以最恰当的选项是until。

4.104 Gift packaging will be_______for all items in our store without anyadditional charge during the Christmas rush.A.availableB.presentableC.receivableD.enjoyable正确答案:A解析:句中说,客人们即将“可以使用”“包装礼品”的服务。

托业模拟考试题及答案

托业模拟考试题及答案

托业模拟考试题及答案1. 根据以下对话,选择最合适的选项完成句子。

A: "I'm looking for a new job."B: "What kind of position are you interested in?"A: "I'm interested in a position that involves a lot of ______."A) travelingB) sleepingC) cookingD) reading答案:A2. 阅读以下句子,选择正确的单词填空。

The company is expected to ______ its profits by 20% this year.A) enhanceB) reduceC) increaseD) decrease答案:C3. 听以下录音,选择正确的答案。

What is the woman's opinion about the new policy?A) She thinks it's too strict.B) She thinks it's fair.C) She thinks it's too lenient.D) She has no opinion.答案:B4. 阅读以下图表,回答问题。

The chart shows the sales figures for three different products over a year. Which product had the highest sales in the third quarter?A) Product AB) Product BC) Product CD) None of the above答案:B5. 根据以下段落,选择正确的答案。

The company has decided to invest in new technology to improve efficiency. This decision was made after careful consideration of the costs and benefits.What was the main reason for the company's decision?A) To increase salesB) To improve customer satisfactionC) To improve efficiencyD) To reduce costs答案:C6. 听以下对话,选择正确的答案。

大学托业阅读分类模拟题(带答案)

大学托业阅读分类模拟题(带答案)

托业阅读分类模拟题PART 1Directions: A word or phrase is missing in each of the sentences below. Four answer choices are given below each sentence. Select the best answer to complete the sentence. Then mark the letter A, B, C, or D on your answer sheet.1、Many economic experts predicted that the of retail sales are expected to climb 2percent in March,A. exposureB. entriesC. mostD. majority2、Once the proposal is approved, the grant will be released three installments.A. byB. fromC. inD. with3、At the awards ceremony, Scott Donie expressed his for the hard work and creativity shown by The Employee of the Year, Cody Erin.A. respectB. respectedC. respectingD. respectable4、The Peach Company has produced the highest quality computers since 1990.A. enormouslyB. lightlyC. consistentlyD. brieflyIt is difficult to be successful in business without some kind of advantage.A. competingB. competitionC. competitiveD. competitivelyAll employee disputes be filed with Grant Hall in human resources, and his team will arrange meetings to resolve the conflict.A. couldB. shouldC. mightD. wouldThe weekly sales meeting w川be held on Monday 10 o'clock in the Convention Center.A. byB. onC. atD. within[解析]词汇:介词1 .这是一道词汇题,选项均由介词构成。

托业(阅读)模拟试卷60(题后含答案及解析)

托业(阅读)模拟试卷60(题后含答案及解析)

托业(阅读)模拟试卷60(题后含答案及解析) 题型有:1. 完成句子 2. 短文填空 3. 阅读理解完成句子1.______you want to receive additional information regarding the services we offer, please log onto our website at www. today.A.IfB.ForC.DespiteD.Whether正确答案:A解析:横线处需填入在句首引导状语从句的从属连词。

(B)for和(C)despite 为介词,故排除。

(D)whether与主句(please log onto…today)在语意上不连贯。

因please log onto…today为“请……”的祈使句,加上从句是有关特定条件的描述,故(A)if最恰当。

2.Sandy Duncan was handpicked by the genere manager to head the next project because of ______experience in this field.A.herB.hersC.herselfD.she正确答案:A解析:横线处需填入修饰名词experience的形容词性物主代词,故选(A)。

若备选答案为人称代词的不同形式,可根据上下文结构,直接选择其适当形态,以节省解题时间。

3.The changes made in the working conditions at our company resulted in remarkable improvements in all aspects of our business and______more so in the morale of our employees.A.allB.anyC.evenD.although正确答案:C解析:横线后的more在解题时起决定性作用。

托业(阅读)模拟试卷49(题后含答案及解析)_0

托业(阅读)模拟试卷49(题后含答案及解析)_0

托业(阅读)模拟试卷49(题后含答案及解析) 题型有:1. 完成句子 2. 短文填空 3. 阅读理解完成句子1.Unless plenty of visitors express their interests, we may not treat_______ to a supper with the author of this soon-to-be-released book.A.theyB.theirC.themD.themselves正确答案:C解析:treat是及物动词。

需要填入宾语。

选项中宾格形式的是them。

themselves 也可以做宾语,但和这里动作的主体we不一致。

2.This problem is______his ability and I don’t think he can solve it.A.toB.inC.beyondD.under正确答案:C解析:这句话的意思是说,这个问题超出了他的能力,我认为他解决不了。

四个选项中,只有beyond可以表示“(在能力、程度上)超过,为……所不能及”,“beyond one’s ability”表示“超出某人能力范围”。

3.Please wait _______ the elevator door to close automatically in order to conserve energy.A.ofB.forC.byD.until正确答案:B解析:知道wait for这个固定搭配就能答对此题。

锦上添花,再多记~点:这里for后面的词是to不定式的主语。

4._______ enjoying talking on the telephone surely finds the equipment convenient.A.ThoseB.EverywhereC.WhoeverD.Anyone正确答案:D解析:空格内应填入整个句子的主语。

托业(阅读)模拟试卷40(题后含答案及解析)

托业(阅读)模拟试卷40(题后含答案及解析)

托业(阅读)模拟试卷40(题后含答案及解析) 题型有:1. 完成句子 2. 短文填空 3. 阅读理解完成句子1.Sandra’s method proved to be ______ in handling multiple tasks at once.A.efficiencyB.efficienciesC.efficientD.efficiently正确答案:C解析:形容词位置2.Please note that ______ personal checks nor credit cards are accepted at Crawford House Catering.A.neitherB.eitherC.bothD.whether正确答案:A解析:关联连词neither A nor B(既不是A,也不是B)3.You will not receive the bonus payment ______ the end of the next fiscal year.A.duringB.untilC.asD.since正确答案:B解析:时间介词until(到……为止)4.Anyone planning to ______ in this workshop should contact Jenny Forbes at 021-555-908 before June 2.A.participateB.attendC.registerD.complete正确答案:A解析:participate in参加(=attend)5.The president will meet with several important dignitaries during his official ______ to Hong Kong next month.A.visitB.visitingC.visitedD.visitor正确答案:A解析:名词位置,名词优先于动名词6.The customer relations manager received praise for ______ handling stubborn customer complaints.A.generallyB.dependentlyC.skillfullyD.currently正确答案:C解析:skillfully巧妙地:精巧地7.Ms. Wilkinson is always willing to do extra work by ______ if no one else can help.A.sheB.herC.hersD.herself正确答案:D解析:<by+反身代词>(自己)8.The Gold Health Clinic is widely recognized for being ______ to looking after the well-being of all its clients.A.supportableB.dedicatedC.providedD.serious正确答案:B解析:be dedicated to doing致力于……9.WRI Bank’s online service allows customers to see ______ how much money they have in their account.A.carefullyB.particularlyC.correctlyD.exactly正确答案:D解析:exactly(信息)完全正确、精确地10.The new online ordering system is ______ more convenient than the old pen-and-paper catalog.A.substanceB.substantiateC.substantialD.substantially正确答案:D解析:副词位置11.At Go Flight Airline this weekend, you can get a return trip to Paris for two for the same price ______ a single ticket.A.onB.withC.toD.as正确答案:D解析:the same ~as A(如同A一样)12.The recent upsurge in industrial activity should help reduce the Zimbabwean government’s ______ on foreign aid.A.reliesB.reliantC.relianceD.reliability正确答案:C解析:名词位置,reliance(信赖的状态)13.The company ______ very strongly in the third quarter, earning a profit of over $200 million.A.performedB.behavedC.conductedD.encouraged正确答案:A解析:perform执行,完成(任务,工作等)14.The ______ weather conditions meant that Flight WX103 to Amsterdam had to be delayed.A.unfavorableB.infrequentC.contentiousD.purposeful正确答案:A解析:unfavorable不顺利的,不宜的15.______ the CEO’s decision to step down, the company’s share price dropped by nearly 2 percent.A.In order toB.As a result ofC.According toD.By means of正确答案:B解析:结果介词as a result of(由于……)16.Before the start of this year, Ms. Gillard ______ about the possibility of running the company as the CEO.A.does not thinkB.was not thoughtC.will not thinkD.had not thought正确答案:D解析:时态,语态17.If the merchandise does not arrive ______ Friday, please inform the customer service representative about the delay.A.atB.byC.withD.in正确答案:B解析:表示动作完成的时间点的介词by(在……之前)18.The Napier Municipal Government ______ consults with members of the community before making major policy decisions.A.regularB.regulateC.regularlyD.regulation正确答案:C解析:副词位置19.Starmart has over 100 stores ______ located in shopping malls and business districts throughout the state.A.acceptablyB.totallyC.convenientlyD.approximately正确答案:C解析:conveniently便利地20.Several students have complained that the noise from the construction site outside is distracting ______ lectures and tutorials.A.whileB.duringC.towardD.whenever正确答案:B解析:表示时间段的介词during(在……期间)21.Freshco Supermarkets succeeded in ______ 12 new stores in Georgia, Alabama, and North Carolina last year.A.openB.openedC.openingD.opens正确答案:C解析:作介词宾语的动名词22.Rest assured that all personal details collected by AMI Insurance will be kept ______ on the company’s private database.A.deliberateB.appreciativeC.diverseD.secure正确答案:D解析:secure安全的,有把握的23.The ankle injury that tennis star Basil Muller sustained yesterday may ______ him from taking part in the Austrian Open next week.A.relieveB.exceedC.conflictD.prevent正确答案:D解析:prevent阻止(prevent A from doing阻止A做某事)24.Information brochures are ______ available for all tourists in hotel lobbies, visitor centers, and airport terminals.A.readilyB.preciselyC.busilyD.formerly正确答案:A解析:readily容易地25.Doctors say that people can dramatically reduce their risk of heart disease by quitting smoking and adopting an ______ lifestyle.A.actingB.activeC.actionD.actively正确答案:B解析:形容词位置,active(活跃的)26.The Walter Foundation has received over $40,000 in donations from local residents since it ______ its fundraising campaign.A.beginsB.beganC.has begunD.will begin正确答案:B解析:<现在完成时+since+过去时态>27.Medibank now offers customers greater ______, allowing them to customize their health insurance policy.A.returnB.flexibilityC.insightD.effect正确答案:B解析:flexibility灵活性,弹性28.Kelly Waterson ______ Jenny Jones on yesterday’s interview panel because Ms. Jones had to handle some urgent business in Prague.A.replacedB.replacingC.replacementD.replaces正确答案:A解析:动词位置,过去时态29.There are lots of interesting tourist sites, souvenir shops, and restaurants ______ walking distance of the conference center.A.onB.atC.acrossD.within正确答案:D解析:within walking distance of从……走不远就到了30.Hellfire director Steve Brock won a National Film Commission Award, and ______ did the film’s main actor, May Hong.A.sameB.eitherC.soD.too正确答案:C解析:<so+助动词+主语>倒装句31.The senior manager, Mr. Valerie, ______ all of the employees filled in their timesheets accurately and submitted them to the payroll division.A.ensureB.ensuringC.ensuredD.to ensure正确答案:C解析:动词位置,时态,数值一致32.Most consumers found it impossible to ______ bottled water from tap water during the experimental taste tests.A.anticipateB.interpretC.distinguishD.acknowledge正确答案:C解析:distinguish区分(distinguish A from B区分A与B)33.All of the products at Wally’s Discount Furniture require ______, but detailed instructions and essential tools are provided.A.assemblyB.supportC.referenceD.development正确答案:A解析:assembly装配34.Passengers may retrieve their items from the overhead locker ______ the safety belt sign has been switched off.A.meanwhileB.underC.althoughD.once正确答案:D解析:条件连词once(一旦……)35.The Food Service Workers’ Union is an organization that helps individuals ______ in the hospitality industry.A.employB.employedC.employingD.employment正确答案:B解析:形容词位置,被动的过去分词36.Watson Beachside offers members of its Elite Club fantastic services and great discounts ______ in its network of resorts.A.altogetherB.everywhereC.beyondD.alongside正确答案:B解析:everywhere到处,各处37.Among the job candidate’s list of relevant ______ are a Masters in Planning and a Graduate Diploma in Management.A.qualifiedB.qualifyingC.qualifiesD.qualifications正确答案:D解析:名词位置38.The developers at Virtech have made several ______ to the LTO Smartphone based on feedback from customer focus groups.A.convictionsB.modificationsC.accreditationsD.considerations正确答案:B解析:modification修改,改变39.One member of the board expressed ______ about the company’s plan to expand into Southeast Asia.A.reservationsB.possibilitiesC.responsesD.qualities正确答案:A解析:reservation保留(意见)40.As a leader, Ms. Gomez is always ______ to new ideas and suggestions from members of her team.A.willingB.receptiveC.approachableD.comprehensive正确答案:B解析:receptive能容纳的短文填空From: Tom EvansTo: All attendeesSubject: Conference programDear attendees, Thank you very much for (81) the 3rd Annual Trade Marketing Conference. This promises to be an exciting and informative event, and I look forward to seeing you all there. One of the highlights of the conference will be the keynote address by internationally-renowned trade marketing guru Gus Hobson. (82) his presentation, you will all have the opportunity to speak to him during our open Question and Answer Forum. In addition, there will be a number of other stimulating talks. I (83) a full copy of the program for your interest, outlining all session times and breaks. Please take the time to look over these details as soon as you can. Regards, Tom Evans Conference Manager41.A.reaching out toB.signing up forC.looking back onD.moving away from正确答案:B解析:sign up for注册,报名参加42.A.FollowingB.LaterC.FurtherD.Considering正确答案:A解析:时间介词following(在……以后)43.A.be attachi’ngB.would attachC.have attachedD.was attached正确答案:C解析:时态,语态Property Market Downturn Over the first quarter of (84) year, there has only been $50 million of turnover in the local real estate market. This compares unfavorably with the $90 million recorded at the same time last year. Beverley Hauritz, the chair of the Professional Realty Association, told reporters that she was not surprised by the figures, which she believes are a result of the global financial crisis. (85) , she is anticipating an even bigger decrease next quarter, as investors look to reduce spending even further. Although there has been talk among (86) that property prices may continue falling throughout the rest of the year, Ms. Hauritz remains confident that the property market will begin to pick up again in the future. Her prediction is that prices will return to their previous levels by the end of the year.44.A.thatB.eachC.everyD.this正确答案:D解析:指示形容词this45.A.HoweverB.In factC.NeverthelessD.Even so正确答案:B解析:副词短语in fact(事实上,实际上)46.A.analysisB.analysesC.analyticalD.analysts正确答案:D解析:名词位置,analyst(分析者)The Formica Family and Community Club is proud to bring you the annual Formica Season of Fun. Beginning on February 3 and finishing on February 23, the Season of Fun provides local residents with a fantastic (87) of exciting and educational events. The program includes a Teddy Bears’ picnic at Holden Park, an Adventure Playground Challenge at the same venue, free street theater in the central city, and a Community Fun Run through the Vira Marine Reserve. (88) of the scheduled events will be open to all-comers. However, those interested in the playground challenge will need to register in advance because space is limited. To find out more about the program for this upcoming festival, visit the Formica Family and Community Club website and (89) on the link to the Formica Season of Fun. We look forward to seeing you all there!47.A.lineB.totalC.approachD.range正确答案:D解析:a range of一系列48.A.MostB.AllC.SomeD.One正确答案:A解析:否定代词most(大部分)49.A.clickB.clickedC.clickingD.clicks正确答案:A解析:动词位置,表同位连词andMr. Matthew PalmerElevit Community CollegePO Box 114Brandeis, WA4December 2010Dear Mr. Palmer, I’m writing in regard to the order you made with us on November 29. Our client information records indicate that you (90) a dozen new desktop computers and a dozen 17-inch LCD monitors. Before we can make this shipment, you will need to pay a deposit of 5% of the value of the order, which comes to $600. You can submit this payment by placing a check in the addressed envelope (91) with this letter. Once this deposit has been processed, will authorize the shipment. The remainder of the bill is (92) to our account on receipt of the goods. For questions or concerns about the billing process,*********************************************************.Kind regards, Tim Gavins Sales Manager —Computer Wholesaler 50.A.refundedB.returnedC.requestedD.recalled正确答案:C解析:request请求51.A.sealedB.separateC.enclosedD.complete正确答案:C解析:enclosed(信封内)附上的52.A.to payB.to be paidC.payingD.being paid正确答案:B解析:表示义务的<be to用法>,语态解题关键句阅读理解To: Jenny Wong Greetingsl According to our records, it’s time for your annual vision checkup. Yearly checkups are recommended to ensure that your glasses prescription has not changed. Wearing glasses with an improper prescription can leadto headaches, eye strain, and further vision loss. As you know, Thompson Vision Care operates three clinics around the city. Please call us to set up an appointment for the time and place mosi convenient to your schedule. We look forward to your visit. - The Thompson Vision Care team -53.Why did Ms. Wong receive this note?A.To thank her for her continued businessB.To inform her of the results of a medical testC.To remind her to schedule an appointmentD.To ask her to pick up her new glasses正确答案:C54.What is indicated about Thompson Vision Care?A.It has more than one location.B.It specializes in treating eye strain.C.It offers annual checkups for free.D.It is a new clinic in the city.正确答案:AFrom: Admin departmentTo: All employees Don’t forget that this Friday is the company’s 15th anniversary celebration. The event will be held downtown at the Farmers Auditorium, 205 Main Street, starting at 6:15 pm and lasting approximately three hours. Due to limited parking options downtown, the company will be operating shuttles from our offices to the auditorium and back. You do not need to register for the shuttle—there will be plenty of room for everyone. Buses will leave from outside the main building entrance starting at 5:00, and after the ceremony they will bring everyone back. Please direct all questions to the Administration Department. See you on Friday!55.Where will buses take employees after the event?A.To a parking garageB.To the auditoriumC.To the office buildingD.To the anniversary celebration正确答案:C56.Which information is NOT provided about the event?A.Its starting timeB.Its locationC.The menu for dinnerD.The means of transportation正确答案:CFrom: Seth Anderson, Head of SalesTo: Sales teamSubject: Survey results Team, we have received the results from our most recent customer survey. What follows is a brief overview of the key findings: * Our most popular product by far is still the Steelman Professional wristwatch. Customers love its sleek, sophisticated look, and they also report that the digital features are easy to program and extremely useful. * We are also getting good feedback about the new Timekeeper wristwatch. Most people who responded to the survey regarding this model pointed to its reliability as the main reason why they liked it. * Despite our best efforts, the Steadfast Super Sport watch still isn’t selling. Customers say that it has so many functions that it is difficult to use. When selling this watch, I recommend that you emphasize its durability instead of its functionality.57.What is the purpose of the email?A.To get feedback on a customer surveyB.To introduce a new productC.To explain better sales techniquesD.To summarize a set of data正确答案:D58.What do customers think of the Timekeeper watch?A.They feel it should be more durable.B.They find its functions difficult to use.C.They appreciate that it is dependable.D.They like its sophisticated design.正确答案:CItinerary —Travel Plus Tour CompanyTour: City Highlights Tour, BarcelonaDate: April 24, 2011 All tours begin at the Travel Plus Tour office, on the first block of La Rambla. Please arrive at least 15 minutes prior to the first time listed on this itinerary.59.When should tourists arrive at the tour office in the morning?A.8:00B.8:15C.8:30D.10:00正确答案:A60.Which is true about Parc Guell according to the itinerary?A.It is near the tour office.B.It features artwork.C.It is surrounded by skyscrapers.D.It is modeled on the Sagrada Familia.正确答案:B61.What is NOT mentioned about the tour?A.Participants must pay for lunch themselves.B.Only a selection of Picasso’s work will be viewed.C.Food and beverages are prohibited in the gothic cathedral.D.Tour members must return to the company’s office at the end.正确答案:DDrimbal Information Systems is an exciting place! As usual, there are many events, updates, and opportunities taking place in the coming month. Below are a few that pertain to all employees. Please notify Gall McPherson in Human Resources if you have any questions. * The company’s annual softball tournament is scheduled for the weekend of June 11. Due to an increase in interest last year, we’re going to be requiring everyone who wants to play to register. This cannot be done online—you must fill out the proper form and deliver it to Bob Billing, the event coordinator, at least one week before game day. Forms can be found in Mr. Billing’s office. * As Drimbal continues to grow, we are taking on more and more international contracts. With this comes the need to modify our business practices to take into account the needs of foreign clients. Helen Daniels from sales will be giving a lecture on this topic on Tuesday, June 14, in Conference Room B. Everyone is encouraged to attend. * And finally, you’ve probably noticed that the employee website has been updated. New features include recent sales figures, upcoming building maintenance information, and a quote of the day from our CEO. We’d love to hear what you think—share your feedback with Arthur Mainz in IT!62.Why was this notice mainly created?A.To inform the staff of the month’s eventsB.To report the results of a softball tournamentC.To introduce newly hired staff membersD.To encourage workers to use the new website正确答案:A63.Who is most likely an expert on foreign business relations?A.Arthur MainzB.Bob BillingC.Gail McPhersonD.Helen Daniels正确答案:D64.What is mentioned about softball registration?A.It will begin the weekend of June 11.B.It is recommended to be made online.C.It should be submitted a week in advance.D.It is being organized by Human Resources.正确答案:C65.What is NOT found on the updated website?A.Scheduling details for repair workB.Feedback from employeesC.Inspiration from the CEOD.Updated company sales information正确答案:BTo: Lee BrightFrom: FiilmoreBankDear Mr Bright: Thank you for your email dated Oct 14 regarding a fee charged to your checking account In answer to your question, the fee of $25 was charged because your average daily balance for the month of September dropped below $5,000 This is in line with the policy that governs your account, which is a Privilege Plus account To avoid this fee in the future, please maintain an average daily balance of $5,000 in the account Or, you can switch to a different type of checking account The Privilege Basic account, for example, has no minimum balance requirement Of course, it also does not entitle you to the benefits of Privilege Plus status: free checks and debit cards, enhanced online security, and online banking services at no charge If you would like to make a change, you can contact a customer service representative at 555-2904 Also, because you are such a valued customer, we will go ahead and refund the fee in question However, this is a one-time action; in the future, any fees will be applied to your account So we encourage you to take the necessary steps to ensure that your account remains in good standing As always, thank you for banking with us66.Why was this email written?A.To respond to a customer inquiryB.To announce a policy changeC.To advertise to a new customerD.To apologize for an error正确答案:A67.What is mentioned about Privilege Basic accounts?A.They aren’t able to be used with checksB.They are more popular than Privilege PlusC.They require customers to pay monthly feesD.They aren’t penalized for low balances正确答案:D68.What does the bank agree to do?A.Change Mr Bright’s account typeB.Provide Mr Bright new checks at no costC.Give Mr Bright another chanceD.Alert Mr Bright prior to charging more fees正确答案:CThis past Thursday, more than 400 people showed up to the Stinson Auditorium for Professor John Wallace’s speech. Unfortunately, Professor Wallace wasn’t among them. It was not until after everyone had taken their seats in the auditorium that they learned the professor would not be able to make it. Early reports indicate that a scheduling mix-up kept the professor from making his appearance. At the time when he was supposed to be speaking to a crowd here in Stinson about literary history, he was giving a lecture on modern journalism at an event in Pittsburgh. Representatives for Professor Wallace have issued multiple apologies to the people of Stinson, assuring them the error was an accident. Thus far, it appears as if they have forgiven him. Tickets for his rescheduled Stinson appearance, this coming Saturday afternoon, have already sold out. It is likely that attendees are most excited to hear the professor talk about his latest bestselling novel, The Artificial Writer. But he has made it clear that this is not a press tour. Instead he will stick to his planned talk on the history of the novel in Western literature.69.What is the article mostly about?A.A disturbance on a college campusB.The rescheduling of a speaking eventC.A recently released book about writingD.The reason a planned book signing was canceled正确答案:B70.What is likely to happen on Saturday?A.Four hundred people will travel to Pittsburgh.B.No one will show up to see Professor Wallace.C.People from Thursday’s event will demand a refund.D.Professor Wallace will speak to a full auditorium.正确答案:D71.What did Professor Wallace talk about in Pittsburgh?A.Current journalistic issuesB.The development of the Western novelC.A historical aspect of literatureD.His latest novel正确答案:A72.What is implied about Professor Wallace?A.He is unapologetic about his mistake.B.His speeches are in high demand.C.He enjoys speaking about his books.D.He did not want to speak at the Stinson Auditorium.正确答案:BZiv Yolis5904 Oakdale WayDallas, TXDear Mr. Yolis, Thank you for your order placed on March 2nd, which included two paperback murder mysteries by author Anne Stevenson. Our records indicate that this was the 20th order you’ve placed wJth us here at Mail-Away Books! We can’t thank our loyal customers like you enough, and we hope you’ll accept the enclosed gift of the crime thriller A Dangerous Time, by Frederick Sutherland. Enjoy! We’d also like to extend to you an offer 1o join the Mail-Away Books Champion Customer Club. Membership is entirely free for you—only our best customers are invited to join! As a member, you’ll receive special discounts on bulk orders and advance notification of upcoming book releases. Plus, you’ll be entered Jnto our annual Champion Customers Club raffle, for a chance to win a $500 gift certificate to Mail-Away Books! All you have to do to join is fill out and mail back the form enclosed. We hope you’ll consider becoming a Mail-Away Books Customer Champion! Sincerely, Deborah DeLoux Founder, Mail-Away Books73.What is the purpose of the letter?A.To offer Mr. ~olis a gift certificateB.To confirm Mr. Yolis’s recent orderC.To inform Mr. Yolis that he has placed more than 20 ordersD.To invite Mr. Yolis into a customer loyalty program正确答案:D74.Which author’s book will accompany this letter?A.Anne StevensonB.Deborah DeLouxC.Ziv YolisD.Frederick Sutherland正确答案:D75.What do Champion Customer Club members NOT receive?A.A collection of mysteriesB.Previews of new booksC.Large orders at lower pricesD.A spot in a raffle drawing正确答案:APerfect community living is here, and its name is Green Cliff! Construction has been slow, but we wanted to get everything just right. And now we have 58 units of stylish, modern living spaces available in a variety of sizes and rent levels. You have to see them to believe them! Our pre-registration period ends today, May 1. Over half of the apartments are already filled, and to show our gratitude to these early leasers, the Green Cliff community is throwing a public barbecue party on Friday, May 15. There will be free food, drinks, and even a temporary swimming pool that we’re renting for the day! For those of you who are thinking about moving into Green Cliff but haven’t decided yet, this is the perfect opportunity for you to come see what it’s all about! The party runs from noon to 6:00 pm and is open to the public. We know you’ll be impressed by this complex and community like none other. Our central garden offers a place for residents to get together and relax in the shade of tall palm trees. The community gym is free of charge for resident use and features all of the most modern exercise equipment. And our security can’t be beat! Still not convinced? Sign a lease before May 31 and we’ll pay your first month’s stay What are you waiting for?76.Where would the information most likely appear?A.In a city planning manualB.In a home design magazineC.In an advertising brochureD.In a book of coupons正确答案:C77.In paragraph 1, line 2, the word “just” is closest in meaning toA.fairlyB.exactlyC.onlyD.nearly正确答案:B78.What is said about the event on Friday?A.It will last all day.B.Advance registration is required.C.It is open to all community members.D.Organizers have not finished planning it yet.正确答案:C79.What is NOT listed as a feature of Green Cliff?A.A community security systemB.A resident-only swimming poolC.A contemporary design styleD.A central meeting area正确答案:B80.What is offered to those who secure a lease before the end of the month?A.Lower monthly paymentB.Homemade barbecueC.One free month of rentD.An exercise machine正确答案:CDate: 01/15/2011From: Edna PerthTo: Alex YoungSubject: Facilities at the GrandAlex, As requested, toured the facilities at the Grand International Hotel a week ago. Their two first-floor ballrooms should be large enough to accommodate the main conference events. Also, just down the hall are several smaller meeting rooms where we can stage the panels and seminar discussions. Unfortunately, our options for dining are not as conveniently positioned. The hotel runs an open-air courtyard cafe just outside the main ballrooms, but there isn’t much seating space—only about 100 chairs. A more spacious and elegant choice would be the Grand International Restaurant, but it’s on the third floor, on the other side of the hotel. I’ll leave it to you to decide which location would be best for the introductory luncheon on the first day. And, of course, I still haven’t heard when exactly the conference will be held. Haven’t you picked a weekend yet? As soon as you do, please contact Regina Lim, the events coordinator at the Grand International. Her number is 555-6392. Edna Date: 01/24/2011 From: Alex Young To: Edna Perth Subject: RE: Facilities at the Grand Edna, thanks for your email, and I apologize for my delay in getting back to you. To answer your last question—yes, a date for the conference has been set. It will run from Friday, February 16, to Sunday, February 18. I’ve already called Regina, and everything is arranged. As for Friday’s luncheon, I believe the outdoor dining area will be adequate. We don’t expect more than 80 participants to be there for the introductory event. Then, for the group lunch on Sunday, we can use the hotel’s main restaurant. Have you started preparing your presentation? I just spoke with Bill Waters, who’s giving the keynote address on Sunday, and he was quite nervous. In fact, I should go help him with it now. See you soon, - Alex81.According to the first email, what is uncertain about the conference?A.Its focusB.Its locationC.Its attendanceD.Its starting date正确答案:D82.What has Mr. Young already done?A.Hosted a luncheon with conference participantsB.Led a tour of the hotel’s facilitiesC.Confirmed the conference date with the hotelD.Prepared a speech for Mr. Waters正确答案:C83.Who works for the Grand International Hotel?A.Bill WatersB.Regina LimC.Edna PerthD.Alex Young正确答案:B84.When was Ms. Perth at the hotel?。

托业(阅读)模拟试卷47(题后含答案及解析)

托业(阅读)模拟试卷47(题后含答案及解析)

托业(阅读)模拟试卷47(题后含答案及解析) 题型有:1. 完成句子 2. 短文填空 3. 阅读理解完成句子1.Your body cannot make up for______ sleep or rest time because it is not physiologically possible.A.losingB.loseC.losenD.lost正确答案:D解析:这句话的意思是说,你的身体不能补偿失去的睡眠或休息时间,因为这在生理上是不可能的。

lost在这里表示”失去的,丢失的”,可知正确项是(D)。

2.But what those placing their hopes on the Chinese______should also care about is whether wages grow or stagnate.A.consumer marketB.consuming marketC.consumption marketD.consumed market正确答案:A解析:这句话的意思是说:不过,那些寄望于中国消费者市场的人们也应当关注该国的工资是增长还是停滞不前。

consumer market意为“消费市场”,为固定搭配,所以正确项为(A)。

3.The publishing company will ask whether you will renew_______ subscription to its magazine or not.A.yourB.yoursC.youD.yourself正确答案:A解析:名词前面如果用代词。

就用形容词性的物主代词。

如果是用于“S+V+IO+DO”句式的动词,经常在做IO的名词后面直接再加名词做DO。

renew 是“更新”的意思。

4.Rebates are_______after purchase at regular sticker prices.A.redeemableC.regretfulD.retrospective正确答案:A解析:be动词后面的形容词修饰主语。

托业(阅读)模拟试卷31(题后含答案及解析)

托业(阅读)模拟试卷31(题后含答案及解析)

托业(阅读)模拟试卷31(题后含答案及解析) 题型有:1. 完成句子 2. 短文填空 3. 阅读理解完成句子1.The accounting department will ______ an internal audit into the company’s expenditure over the last two quarters.A.conductB.conductedC.conductingD.be conducted正确答案:A解析:动词位置,<助动词+动词原形>,语态2.Police officers caught the ______ outside a shopping center on the afternoon of March 14.A.offenceB.offensiveC.offendD.offender正确答案:D解析:名词位置,offender(罪犯)3.Ms. Long is preparing for ______ meeting with the executives from Western Technology Institute on Thursday.A.sheB.herC.hersD.herself正确答案:B解析:代词的所有格4.Mr. Bill’s ______ of the job offer at the Department of Defence is dependent on his receiving a security clearance.A.acceptB.acceptableC.acceptedD.acceptance正确答案:D解析:名词位置5.The independent report on government infrastructure contains ______ reviews of the road and rail networks.A.promptB.capableC.extensiveD.spacious正确答案:C解析:extensive广泛的6.______ the start-up venture is risky, Mr. Skinner is still willing to invest $2 million of his own money in it.A.Even thoughB.DespiteC.BecauseD.Whenever正确答案:A解析:让步连词even though(即使……)7.It looks as though Ms. Poidevan will not ______ the company at the tradeshow because she is feeling unwell.A.representB.demonstrateC.expandD.preserve正确答案:A解析:represent(代表)8.The law prohibits hospitals and medical practices from distributing a patient’s ______ information to any third party.A.personB.personalC.personallyD.personality正确答案:B解析:形容词位置9.Although some bus drivers have signed the collective pay agreement, thereare many ______ are refusing to do so.A.whatB.whichC.whoD.when正确答案:C解析:关系代词who作先行词代替人10.Several important diplomats were ______ the guests attending the president’s 50th birthday celebrations at his home in Bolivia.A.priorB.amongC.throughoutD.selected正确答案:B解析:地点介词among(在……中间)11.With Netwise Bank, you will not be charged any additional fees for online banking ______.A.deficitsB.transactionsC.expensesD.providers正确答案:B解析:transaction(经营,业务等方面的)交易12.Sales of our car models have risen by around 50 percent ______ to the results last quarter.A.matchedB.evaluatedC.comparedD.weighed正确答案:C解析:compare比较(compared to与……相比)13.______ attending a conference on regional development today, the mayor will hold a press conference to discuss the election.A.According toB.Seeing asC.Instead ofD.Due to正确答案:C解析:介词instead of(代替……)14.The findings from our market research are ______ unexpected that we must conduct another test to confirm them.A.asB.tooC.soD.very正确答案:C解析:<so+形容词[副词]+that从句>(由于太……,结果……)15.Ms. Suarez ______ for Chair of the Board in December 2009, and took up the position in March of the following year.A.was nominatedB.was nominatingC.had nominatedD.will be nominated正确答案:A解析:过去时态,被动语态16.The hotel management trains all its staff members to deal with guest inquiries ______.A.courtesyB.courteousC.courteouslyD.more courteous正确答案:C解析:副词位置17.The ski season is ______ over, so many people will head up to the mountains this weekend for one last time.A.graduallyB.nearlyC.severelyD.formerly正确答案:B解析:nearly几乎18.The attorney’s ______ to the shipping company is to sue the port authorities for causing a loss of profit.A.judgmentB.recommendationC.activityD.progression正确答案:B解析:recommendation推荐,建议19.Mr. Lyall ______ to the waiter at Merci Beaucoup that his steak had been overcooked.A.congratulatedB.complainedC.resistedD.demanded正确答案:B解析:complain发牢骚20.The Consumers’ Institute website is normally a ______ primary source of information about new products.A.relyB.relyingC.reliableD.reliably正确答案:C解析:形容词位置,形容词优先于分词21.______ scientists to develop a vaccine for the bird flu virus, they could potentially help save millions of lives.A.IfB.ShouldC.WereD.Unless正确答案:C解析:were to虚拟语气倒装句22.At the end of the tour, travelers are required to meet ______ the front steps of the hotel to be picked up.A.atB.inC.toD.out正确答案:A解析:地点介词at(在某处)23.______ next month, employees will have to log in on the company intranet when they arrive at work.A.StartingB.HavingC.TakingD.Doing正确答案:A解析:表示开始的介词starting(开始……)24.It remains to be seen ______ Virotech’s decision to acquire BHI Industries will result in increased profits.A.whetherB.in order thatC.whereasD.that正确答案:A解析:名词性从句连接词whether(是否……)25.Entries for the design competition will be accepted ______ the month of April.A.atB.alongC.throughoutD.while正确答案:C解析:表示时间段的介词throughout(自始至终)26.It is expected that HX Industrial Bank will ______ become the biggest player in the local banking market.A.eventualB.eventuallyC.eventuateD.event正确答案:B解析:副词位置27.The proposed reduction in interest rates will ______ influence the housing market and other key economic areas.A.adequatelyB.positivelyC.previouslyD.exactly正确答案:B解析:positively肯定地28.The beachside resort only ______ capacity in the busy vacation period from the end of May to the beginning of August.A.setsB.reachesC.makesD.provides正确答案:B解析:reach到达,抵达29.The latest figures show that the unemployment rate is at its ______ level since the oil crisis of the 1970s.A.highB.highlyC.higherD.highest正确答案:D解析:最高级30.The news about the deadly terrorist attacks in Dubai ______ people around the world.A.astonishedB.incorporatedC.exclaimedD.remarked正确答案:A解析:astonish使惊讶31.Goods and services are less ______ in North Carolina and Mississippi thanthey are in other parts of the country.A.assumedB.proficientC.apparentD.extravagant正确答案:D解析:extravagant奢侈的32.It is very difficult for non-governmental organizations to measure the ______ of the aid they supply to developing nations.A.findingB.satisfactionC.accuracyD.effectiveness正确答案:D解析:effectiveness效力33.The company keeps ______ on all business deals, accounting processes, and employment matters.A.documentationB.encouragementC.dependenceD.institution正确答案:A解析:documentation文件材料34.Even if you have a fear of public speaking, you should still force ______ to make presentations.A.youB.yourC.yoursD.yourself正确答案:D解析:反身代词35.______ from Angelica Diaz, none of the conference presenters had previous experience speaking at big industry events.A.AlongB.AwayC.AsideD.Ahead正确答案:C解析:介词aside from(除……以外)36.The new CEO’s unconventional approach to the job ______ some members of the board of directors.A.concernB.concernsC.concerningD.is concerned正确答案:B解析:动词位置,数值一致,语态37.CFD Packing and Moving wraps each of your household items individually to ensure they are not damaged ______ transit.A.withB.inC.onD.for正确答案:B解析:in transit在途中38.The airline announced that it ______ refunds to customers because of delays caused by inclement weather.A.will not have been offeringB.will not be offeredC.would not be offeringD.would not be offered正确答案:C解析:时态一致,语态39.The manager of the grocery store ______ positioned the snacks and candy bars near the cash register.A.supposedlyB.strategicallyC.analyticallyD.formidably正确答案:B解析:strategically战略性地40.The hiring committee will make the final ______ about who to appoint as the new purchasing director.A.authorizationB.determinationC.consentD.conviction正确答案:B解析:determination决定短文填空Michelle Stevens4121 Moss St.St. Petersburg, FL 59340Dear Valued Banking Customer: We here at Florida National Bank have always promised to put you and your banking needs first. Now, we are happy to introduce a policy change that we think (81) the international transactions of our customers. Starting next month, we will begin (82) all bank fees on both incoming and outgoing wire transfers for international banking transactions. In the current economic climate, we believe it is wise to encourage our clients to deal internationally, not discourage them. This fee cancellation should do just that, and it’s another example of how we’re working (83) our customers. Thank you for your continued support, Dick Johnson CEO, Florida National Bank41.A.has facilitatedB.will facilitateC.facilitatingD.had been facilitating正确答案:B解析:将来时态42.A.waivingB.handlingC.doublingD.deleting正确答案:A解析:waive放弃(权利,主张等)43.A.to satisfyB.satisfactorilyC.satisfyingD.satisfaction正确答案:A解析:不定代词to的副词性用法The World Hunger Council is committed to doing all we can to fight the problems of hunger and poverty in our world. To this end, we are putting on a “Race Against Hunger” event on the weekend on August 21 st. The primary (84) of this event is to raise money for the council’s upcoming charity drive for Africa, where we will be partnering with the organization Food Relief to deliver food supplies to rural African villages. We (85) with Food Relief many times in the past and are quite confident in their ability to help us in this endeavor. Anyone interested in participating in the race should register at . Also, all racers will be required to raise $150, 100% of which will go towards ending hunger in Africa. Questions about the event can be directed to Pam Smith, charity (86) for the World Hunger Council, at 555-2940. Thank you for fighting with us to make the world a better place, and see you at the race!44.A.shotB.concernC.signD.goal正确答案:D解析:goal目标45.A.will cooperateB.cooperatesC.would be cooperatingD.have cooperated正确答案:D解析:现在完成时46.A.coordinatedB.coordinationC.coordinatingD.coordinator正确答案:D解析:名词位置,coordinator协调者Welcome to Camden Provincial Park, one of the country’s oldest and most celebrated natural spaces! The 745 acres of the park were (87) to the provincial government by entrepreneur and naturalist John Camden in 1899. Since then, the area has been instrumental in the preservation of the yellow-petaled sprite flower and the western brown shrub bush. These species grow (88) in this region and no other in the world. In modern times, the park has added recreation facilities to encourage people to visit and appreciate this unique landscape. Over 25 miles of hiking trails await you, and there are 16 primitive campsites available to guests. Please remember to follow (89) park rules at all times, and enjoy your time in Camden Provincial Park!47.A.programmedB.grantedC.assumedD.revealed正确答案:B解析:grant授予48.A.evenB.quiteC.enoughD.only正确答案:D解析:副词only(仅仅)49.A.postingB.postersC.postD.posted正确答案:D解析:形容词位置From: info@To: sarah.brogen@Subject: RE: reservation requestDear Ms. Brogen, Thank you for your interest in our hotel. You have requested one double room for check-in on June 7 and check-out on June 13. I’m pleased to report that we do have (90) on these dates, and your reservation has been made. When you arrive at the airport in Cuzco, there are two ways to get to the hotel. The first is to arrange your own taxi. Simply tell the driver you are staying at the Inca Hotel on San Bias Plaza. (91) , we can send our own driver to pick you up, for the small cost of $10 USD. If you would like to take (92) of this service, please let us know at least 48 hours in advance. We look forward to your stay with us.50.A.complaintsB.availabilityC.organizationD.guarantees正确答案:B解析:availability实用性51.A.AlternativelyB.SubsequentlyC.ImmediatelyD.Accordingly正确答案:A解析:alternatively非此即彼,二者择一地52.A.advantageouslyB.advantagesC.advantageousD.advantage正确答案:D解析:take advantage of利用阅读理解Dear Residents, The Holt Towers Residents’Association will hold its monthly meeting next Monday evening at 7 pm. The event will take place in the conference room on the first floor of the main building. Please aim to arrive 10minutes in advance, so that the meeting can begin on time. A brief agenda of items to be discussed is included below. -Security: Several residents have expressed concern about security guards being inattentive while on duty. Possible solutions to this problem will be discussed at length. - Playground: The children’s playground in front of the north tower is in need of repair. Attendees will review plans for a renovated playground. - Association fees: The annual review of the association’s fees will take place.53.What are readers asked to do?A.Confirm their attendanceB.Suggest ideas for the agendaC.Get to the venue by 6:50 pmD.Read the minutes of a meeting正确答案:C54.What is suggested about the apartment’s security service?A.It is performing poorly.B.It is currently understaffed.C.It is due for its annual review.D.It is expected to expand in future.正确答案:AMs. Pamela Jenkins145 Hardwood DriveAlton, MA 02187August 15, 2010Dear Ms. Jenkins, According to our records, your subscription to the Alton Herald is due to expire at the end of this month. Unless you renew your subscription within the next two weeks, the newspaper will no longer be delivered to your premises. Subscriptions are all handled online now, so in order to renew, please visit our website at www. . By clicking on the “renew” button in the top right corner and following the simple instructions, you will ensure that you continue to receive a hard copy of the paper every day. Moreover, you will also be entitled to view premium news content on our website, such as exclusive pictures and videos, expert opinion and analysis, and much more. Regards, The Team —Alton Herald 55.What is the main purpose of this letter?A.To double-check a subscriber’s personal detailsB.To remind a subscriber to renew a contractC.To explain a change to the subscription serviceD.To advertise a newly launched newspaper正确答案:B56.What is available via the website?A.Breaking newsB.Current affairs linksC.Email updatesD.Special content正确答案:DLocal dance enthusiasts are in for a special treat this spring. The Jacksonville Ballet Company will perform The End at Polygon Hall from the 1 st of March. The season is set to run for three weeks, ending on Saturday, March 21. As in previous years, tickets to the event are expected to sell out quickly. The End is a sad and absorbing tale written by Russian choreographer and composer Dmitri Valos. Set against the backdrop of World War I, it depicts a tragic love story between a German soldier, Hans, and a young Russian woman, Kristina. Up-and-coming star Nathan Hinds will play the role of Hans in the production, while the role of Kristina has gone to an amateur, local woman Mariana Ostev. Overseeing the production will be experienced director Heather Lynwood. Ms. Lynwood was unavailable to comment on her plans for the production. Although Lynwood is renowned for her unique production style, it is understood that she will adopt a traditional approach for The End.57.What is indicated about the event?A.It is the company’s first production.B.It is expected to be popular.C.It will run until the end of the year.D.It will be warm and uplifting.正确答案:B58.Who is Mariana Ostev?A.A dancing starB.A novice ballerinaC.A ballet directorD.A choreographer正确答案:B59.What is NOT mentioned about Ms. Lynwood?A.She has directed ballets before.B.She is famous for her unusual approach.C.She is running her own production agency.D.She wasn’t able to reveal anything about the production.正确答案:CFrom: Glenda HughesTo: AllSubject: Training opportunityHello everyone, I’m sending this email out to inform you all of an excellent training opportunity. The professional development team is holding a special workshop on Friday afternoon to which you are all invited. The session will cover public speaking skills, discussing basic principles and special techniques that can help people improve their ability to deliver presentations. I know that public speaking can be a difficult challenge for many people. If you’re in that category, then this is an excellent chance to work on an area of weakness. The session will be led by Dane Forbes, a former employee of this company who has set up his own consulting business. Dane has delivered hundreds of presentations over the course of his career, so he brings a lot of knowledge and experience to the workshop. There will also be an opportunity for attendees to discuss their own experiences with public speaking. Attendees should bring along a pen and some paper in order to take notes and complete some basic exercises. The workshop will take place in Conference Room 2 from 1:30 pm to 4:30 pm on Friday. I hope to see you there. Glenda Hughes60.Who most likely will attend the workshop?A.Staff members who regularly deal with the publicB.Managers with poor communication skillsC.Workers who need help with writing skillsD.Employees with poor presentation skills正确答案:D61.What is mentioned about the workshop leader?A.He is on the professional development team.B.He has run the workshop hundreds of times.C.He is no longer employed by the company.D.He used to find public speaking challenging.正确答案:C62.What are participants instructed to do?A.Write about their own experiencesB.Take writing materials to the eventC.Register for the workshop in advanceD.Complete some exercises beforehand正确答案:BAttn: Research Staff As part of our initiative to engage more vigorously with our consumers, we have decided to conduct a number of focus groups over the next couple of weeks. Participants in the focus groups will provide feedback on issues such as our branding, our new range of products, and our prices. We hope this research work will generate a lot of ideas that we can then use to improve our products and boost sales. However, in order to run the events smoothly, we need some of you to volunteer your services. The focus groups will run in the evenings, from 7 pm to 9 pm. Given time constraints, we hope to run five tables concurrently each evening. We will need a facilitator and a transcriber at each table, which is where you come in. We hope that you will make the most of this interesting opportunity. The facilitator’s main tasks are to keep the conversation on track and make sure everyone in the group is contributing. Accordingly, facilitators need to have excellent interpersonal skills. Transcribers, on the other hand, will be responsible for recording the focus group participants’ideas and suggestions. Therefore, transcribers require good listening skills and the ability to take lots of notes quickly. If you feel you are suited to either of these roles, please email Jenny Richards at j.richards@ to let her know.63.What is NOT indicated about the focus groups?A.They will be facilitated by employees.B.They will be recorded on devices.C.They will be held in the evenings.D.They will be used to generate ideas.正确答案:B64.How many workers are needed?A.5B.7C.9D.10正确答案:D65.Who might be an appropriate facilitator?A.Someone with previous experienceB.A worker with good people skillsC.A person who can handle complex tasksD.An employee who can take notes quickly正确答案:B66.Why should readers contact Jenny Richards?A.To find out more about the positionsB.To sign up for a specific taskC.To discuss their regular job responsibilitiesD.To nominate colleagues for the available jobs正确答案:BAre your curtains or blinds dusty and dirty? Don’t waste time trying to take care of them yourself. Get professional help instead—call Blinkers! Blinkers is a company that Texans have trusted for over 50 years. Blinkers services homes, offices and even factories, classrooms and laboratories, all for a reasonable price. The company’s experienced and trained professionals will have your blinds looking clean and new again in no time. In addition, Blinkers guarantees service within three working days of the initial request. If your needs are not met within that time frame, the service will be provided completely free of charge.67.What is being advertised?A.A curtain and blind storeB.An upholstery retailerC.A window treatment cleaning serviceD.A repair service for blinds and curtains正确答案:C68.What is NOT said about Blinkers’ service?A.It is very prompt.B.It is of a high quality.C.It is quite affordable.D.It is free the first time.正确答案:DFrom: Gemma Hutchison, GMOTo: Paolo Santos, Hilton FinancialSubject: DeliveryDear Mr. Santos, First of all, I’d like to apologize for the mix-up with your company’s order yesterday. As I understand it, you received 10 ink cartridges fewer than you requested and an additional box of printer paper, as well. At GMO, we pride ourselves on our attention to detail, so I immediately began an investigation. It turns out that a member of the administration team caused the problem. He confused your order with that of another firm with a similar name, Hillman Finance. This sort of mistake is unacceptable, and I have given that employee an official warning about his performance. I can assure you that all staff members will take the utmost care to ensure your order is processed correctly in future. The good newsis that the correct shipment is now on its way. It should arrive sometime tomorrow afternoon. Please make sure a company representative is on hand to sign the delivery invoice at that time. I understand that our error has caused your firm significant distress, since you are now running low on stationery supplies. As compensation, I am willing to offer a 30% rebate on this order, as well as a $50 credit on your next order with us. I hope that these terms are satisfactory. Regards, Gemma Hutchison —GMO69.Where most likely does Ms. Hutchison work?A.A finance companyB.A post officeC.A printing shopD.An office supplies firm正确答案:D70.What led to the mistake?A.A computer malfunctionB.A mix-up over namesC.A shipping delayD.A procedural problem正确答案:B71.What is Mr. Santos asked to do?A.Ensure someone is there to receive the deliveryB.Reenter the details of his request into the ordering systemC.Hand over a copy of the original invoice to the courierD.Sign some documents that will be faxed through正确答案:A72.What will Mr. Santos receive?A.A discount couponB.Free shippingC.Some money backD.Complimentary goods正确答案:CRoral Festival Visitors can now come and enjoy the annual Floral Festival at Gleeson Park. The Floral Festival features beautiful flower gardens with sunflowers, petunias, daffodils and much, much more. As well as gorgeous flower beds, the FloralFestival also has regular special events like music concerts, film screenings, and opera performances. To view a full program, visit www. . Attendance at the Floral Festival is free, though donations will be accepted at the gate. How to Get There By Car Parking in and around Gleeson Park is strictly limited. Visitors are advised not to bring their cars unless completely necessary. By Bus Visitors can catch a bus from anywhere in the city to the downtown terminal. From there, it is a short stroll to Gleeson Park. By Bike Bicycle parking racks are provided at Gleeson Park. The racks are secure and cost nothing to use.73.What is the purpose of the information?A.To publicize a community eventB.To solicit donations for a special causeC.To ask for public feedback on a proposalD.To announce a program change正确答案:A74.What is suggested about Gleeson Park?A.It has ample vehicle parking.B.It is close to the city center.C.It is illegal to ride a bike there.D.It will be turned into a bus terminal.正确答案:B75.What are readers asked to do if possible?A.Avoid driving to the venueB.Collect donations at the gateC.Refrain from damaging the flower bedsD.Make suggestions about the program正确答案:AMuseum Keeping Up with the Times Graham Roux, director of the Lyall Bay Museum, announced today that the facility will undergo a major renovation over the next two years. The museum was initially built in 1974 ancl has received no significant structural upgrades since that time. Mr. Roux unveiled the redevelopment proposal at an official ceremony yesterday. During his speech, he indicated that, in its current state, the museum is too ordinary and lacks notable features to attract local visitors and foreign tourists. Mr. Roux has been working on the project since he was appointed lasl June. Although his ideas quickly generated enthusiasm in the community, raising the funds to carry out the task proved extremely difficult. His initial plan to charge admission to the museum met with fierce opposition. Instead, hefinally managed to secure $20 million dollars from private sources, as well as a $4 million granl from the Lyall Bay Municipal Government, to fund the redevelopment project. The plans for lhe renovated museum are ambitious. There will be a new alrium decorated with traditional arts and artifacts. In addition, the area in front of the main entrance, which is currently a concrete square, will be changed to include a lawn and gardens, creating a pleasant atmosphere for visitors. The plans also show an extended eastern wing. This will house a new exhibit called the “21st Century Collection,” where objects and symbols popular in today’s world will be on show. The aim of the exhibit, Mr. Roux says, is to show visitors that “history is being made every day.” Members of the public can view the plans either at the museum reception desk, or online al www.lyallbaymuseum.corn.76.What is the article mainly about?A.A museum openingB.A facility upgradeC.A public works grantD.An urban renewal project正确答案:B77.According to Mr. Roux, what is currently the major issue of the museum?A.Its design is too modern.B.It looks unimpressive.C.It is in need of repair.D.Its display space is limited.正确答案:B78.How did Mr. Roux raise money?A.Through an increase in the admission feeB.By encouraging companies to invest in the facilityC.Through a combination of public and private donationsD.By holding a range of special fundraising events正确答案:C79.What will be featured in the new exhibit?A.Traditional artifactsB.Contemporary itemsC.Local imagesD.Futuristic visions正确答案:B80.What is NOT mentioned about the plans?A.An atrium will be added.B.A public square will be constructed.C.One wing will be expanded.D.A green outdoor area will be created.正确答案:BRobbie’s Flowers442 Hyde LaneAnderton, Michigan 48169 Order No.: 111902 Date of order: November 14 Customer: Helen Botham Delivery address: 11-16 Carter Plaza Carter Road Anderton, Michigan Contact No.: 231-708-7807 Order Details: 10 dozen red roses $220 10 dozen white lilies $380 8 dozen tulips $210 20 dozen daisies $210 Arrangement fee $460 Delivery fee $65 Value-added tax $152.50$1677.50 Please note that, unless a specific request is made, all deliveries occur the day after the initial order is made. If you require a rush service, and you have not already indicated this preference while placing your order online or over the phone, please contact Robbie’s Flowers immediately at 231-809-8772. For all other inquiries, email robbies@. From: helen.botham@ To: robbies@ Subject: Invoice Hello, I placed an online order through the Robbie’s Flowers website early this morning. I received the delivery just a few moments ago. I am always impressed at your speedy service. I know I can rely on Robbie’s Flowers for last-minute jobs, which is great. However, there was a problem with this particular delivery. You have charged me for an arrangement fee, but all of the flowers came unarranged. This could potentially have been a big problem for me, as I am currently preparing for a big corporate function this evening. Luckily, though, I have a large team of workers here who are able to take care of this task. I hope that I will be compensated for the cost of arranging the flowers. I also expect that you will investigate this situation to find out why the error occurred. Unless you can reassure me of the quality of your service, I will hesitate to use your services in the future. Regards, Helen Botham81.Which item is the cheapest per unit?A.RosesB.LiliesC.TulipsD.Daisies正确答案:D82.What is the purpose of Ms. Botham’s email?。

托业考试阅读模拟试题

托业考试阅读模拟试题

has been a lot of _______ prejudice in American .A) realistic B) racial C) recycled D) rebellious美国一直都有种族偏见。

答案 : racialis an incurable idealist; it's unlike to help him come down to earth and be more _______.A) realistic B) racial C) recycled D) rebellious他是个无可救药的理想主义者;很难将他拉回现实、让他更实际点。

答案 : realistic3. The relations between my mother and brother were getting worse as my brother grew older and more ______ .A) realistic B) racial C) recycled D) rebellious随着哥哥年纪渐长也更叛逆,他与母亲的关系变得更糟。

答案 : rebellious4. Ecological vehicles are made of materials that can be easily _______ once they are dumped.A) realistic B) racial C) recycled D) rebellious环保车使用的材料使它们在被丢弃后还可以轻易地回收使用。

答案 : recycledgoes to that pub every night to have a bottle of beer. He is a _______ customer.A) reliable B) regular C) reluctant D) religiousTom每晚都去那家酒吧喝瓶啤酒。

  1. 1、下载文档前请自行甄别文档内容的完整性,平台不提供额外的编辑、内容补充、找答案等附加服务。
  2. 2、"仅部分预览"的文档,不可在线预览部分如存在完整性等问题,可反馈申请退款(可完整预览的文档不适用该条件!)。
  3. 3、如文档侵犯您的权益,请联系客服反馈,我们会尽快为您处理(人工客服工作时间:9:00-18:30)。
相关文档
最新文档